89
CONTEÚDO AOS LEITORES 2 XXI OLIMPÍADA BRASILEIRA DE MATEMÁTICA 3 Problemas e soluções da Primeira Fase XXI OLIMPÍADA BRASILEIRA DE MATEMÁTICA 13 Problemas e soluções da Segunda Fase XXI OLIMPÍADA BRASILEIRA DE MATEMÁTICA 21 Problemas e soluções da Terceira Fase XXI OLIMPÍADA BRASILEIRA DE MATEMÁTICA 36 Resultados ARTIGOS EQUAÇÕES DIOFANTINAS 39 Antonio Caminha Muniz Neto SOLUÇÕES DE PROBLEMAS PROPOSTOS 49

CONTEÚDO · Web viewA OBM divulga apenas os nomes e pontuações dos alunos premiados; a OBM nunca divulgou nem divulgará comparações entre colégios. Nosso objetivo é estimular

Embed Size (px)

Citation preview

Page 1: CONTEÚDO · Web viewA OBM divulga apenas os nomes e pontuações dos alunos premiados; a OBM nunca divulgou nem divulgará comparações entre colégios. Nosso objetivo é estimular

CONTEÚDO

AOS LEITORES 2

XXI OLIMPÍADA BRASILEIRA DE MATEMÁTICA 3Problemas e soluções da Primeira Fase

XXI OLIMPÍADA BRASILEIRA DE MATEMÁTICA 13Problemas e soluções da Segunda Fase

XXI OLIMPÍADA BRASILEIRA DE MATEMÁTICA 21Problemas e soluções da Terceira Fase

XXI OLIMPÍADA BRASILEIRA DE MATEMÁTICA 36Resultados

ARTIGOS

EQUAÇÕES DIOFANTINAS 39Antonio Caminha Muniz Neto

SOLUÇÕES DE PROBLEMAS PROPOSTOS 49

PROBLEMAS PROPOSTOS 59

AGENDA OLÍMPICA 61

COORDENADORES REGIONAIS 62

AOS LEITORES

Page 2: CONTEÚDO · Web viewA OBM divulga apenas os nomes e pontuações dos alunos premiados; a OBM nunca divulgou nem divulgará comparações entre colégios. Nosso objetivo é estimular

Sociedade Brasileira de Matemática

Realizamos durante 1999 a XXI Olimpíada Brasileira de Matemática em mais de 2.500 colégios de nosso país, atingindo na realização da primeira etapa cerca de 60.000 alunos. Este ano a Olimpíada se realizará nas seguintes datas:

Primeira Fase – Sábado, 10 de junhoSegunda Fase – Sábado, 02 de setembro

Terceira Fase – Sábado, 21 de outubro (níveis 1,2 e 3)Domingo, 22 de outubro (nível 3 - segundo dia).

A Comissão Nacional de Olimpíadas entende que todo aluno que desejar participar da OBM deve poder fazê-lo sem restrições. A comissão oferece inclusive a alunos de escolas que não participam da OBM a possibilidade de fazer as provas sob supervisão direta do Coordenador Regional. As escolas podem naturalmente aconselhar seus alunos a participar ou não da Olimpíada de acordo com seus próprios critérios, mas a escola nunca deve impedir um aluno de participar se este for o seu desejo.

Lembramos que a Olimpíada Brasileira de Matemática é uma competição entre alunos e não entre colégios. A OBM divulga apenas os nomes e pontuações dos alunos premiados; a OBM nunca divulgou nem divulgará comparações entre colégios. Nosso objetivo é estimular o estudo de Matemática entre os jovens, contribuir para o aprimoramento dos professores e propiciar uma melhoria do ensino e do aprendizado desta matéria nas escolas brasileiras e não comparar desempenhos de escolas.

O Regulamento da OBM foi atualizado. Leia o novo regulamento no site:http://www.obm.org.br/regulamento.htm

Finalmente, aproveitamos, para registrar a realização da Semana Olímpica 2000, atividade que vem sendo realizada desde 1998. Nesta oportunidade o evento teve lugar na Universidade Metodista de Piracicaba (UNIMEP) entre os dias 21 a 27 de janeiro de 2000. Durante a Semana Olímpica 2000, reunimos alunos ganhadores da XXI Olímpiada Brasileira de Matemática nos seus três níveis de competição. Estes alunos participaram de um treinamento intensivo com professores de diversas partes do país como preparação para a futura formação das equipes que representarão o Brasil em Olimpíadas Internacionais. Além disso eles tiveram a oportunidade de conquistar novas amizades, iniciando um relacionamento extremamente proveitoso com outros jovens da mesma faixa de idade e com interesses semelhantes.

XXI OLIMPÍADA BRASILEIRA DE MATEMÁTICAPrimeira Fase - Nível 1

EUREKA! N°7, 2000

2

Page 3: CONTEÚDO · Web viewA OBM divulga apenas os nomes e pontuações dos alunos premiados; a OBM nunca divulgou nem divulgará comparações entre colégios. Nosso objetivo é estimular

Sociedade Brasileira de Matemática

01. Um pequeno caminhão pode carregar 50 sacos de areia ou 400 tijolos. Se foram colocados no caminhão 32 sacos de areia, quantos tijolos pode ainda ele carregar?A) 132 B) 144 C) 146 D) 148 E) 152

02. A calculadora de Juliana é bem diferente. Ela tem uma tecla D, que duplica o número escrito no visor e a tecla T, que apaga o algarismo das unidades do número escrito no visor.Assim, por exemplo, se estiver escrito 123 no visor e apertarmos D, teremos 246; depois, apertando T, teremos 24. Suponha que esteja escrito 1999. Se apertamos D depois T, em seguida D, depois T, teremos o número:A) 96 B) 98 C) 123 D) 79 E) 99

03. O gráfico abaixo mostra o valor aproximado do dólar em reais no dia 15 dosúltimos 6 meses.

Marcelo comprou um carro usando um sistema de financiamento chamado leasing corrigido pela variação do dólar e suas prestações vencem exatamente no dia 15 de cada mês. Em dezembro, Marcelo pagou R$ 600,00 de prestação. Com base na tabela, podemos dizer que em maio a prestação foi de:A) R$ 700,00 B) R$ 850,00 C) R$ 650,00 D) R$ 900,00 E) R$ 800,00

04. Numa certa cidade, o metrô tem todas suas 12 estações em linha reta. Adistância entre duas estações vizinhas é sempre a mesma. Sabe-se que a distância entre a terceira e a sexta estações é igual a 3 300 metros. Qual é o comprimento dessa linha?A) 8,4 km B) 12,1 km C) 9,9 km D) 13,2 km E) 9,075 km

05. A metade do número é igual a: A) B) C) D) E)

EUREKA! N°7, 2000

3

Page 4: CONTEÚDO · Web viewA OBM divulga apenas os nomes e pontuações dos alunos premiados; a OBM nunca divulgou nem divulgará comparações entre colégios. Nosso objetivo é estimular

Sociedade Brasileira de Matemática

06. Quantos números de dois algarismos são primos e têm como antecessor umquadrado perfeito ?A) 2 B) nenhum C) 1 D) 3 E) 6

07. Quantas vezes num dia (24 horas) os ponteiros de um relógio formam ângulo reto ?A) 48 B) 44 C) 24 D) 22 E) 23

08. Dona Zizi comprou 2 balas para cada aluno de uma 5a série. Mas como osmeninos andavam meio barulhentos, ela resolveu redistribuir essas balas, dando 5 para cada menina e apenas 1 para cada menino. Podemos concluir que na 5a sérieA) 20% são meninos B) 30% são meninas C) 75% são meninos D) 50% são meninas E) 66,6...% são meninos

09. Vários caixotes cúbicos de plástico Azul ficaram armazenados ao ar livre, na posição indicada na figura ao lado, na qual apenas um dos caixotes não é visível. Com o tempo, o plástico exposto ao ar perdeu sua cor, tornando-se cinza. Ao desfazer a pilha, verificaremos que o número de caixotes com três faces azuis e três cinzentas será:A) 4 B) 5 C) 3 D) 2 E) 1

10. Ronaldo, sempre que pode, guarda moedas de 50 centavos ou 1 real.Atualmente, ele tem 100 moedas, num total de 76 reais. Quantas moedas de um valor ele tem a mais do que a de outro valor ?A) 48 B) 4 C) 8 D) 52 E) 96

11. Juntando três quadrados congruentes e fazendo coincidirlados comuns, sem superposição, podemos formar duas figuras diferentes, como mostra a figura ao lado. Observe que uma figura obtida de outra por rotação, deslocamento ou reflexão, é congruente à mesma figura (muda apenas a posição). Por exemplo, temos abaixo figura iguais em 4 posições diferentes:

EUREKA! N°7, 2000

4

Page 5: CONTEÚDO · Web viewA OBM divulga apenas os nomes e pontuações dos alunos premiados; a OBM nunca divulgou nem divulgará comparações entre colégios. Nosso objetivo é estimular

Sociedade Brasileira de Matemática

Vamos agora pegar três losangos congruentes, um deles representado ao lado. Quantas figuras diferentes podemos formar com os três losangos, fazendo coincidir lados comuns?

A) 1 B) 2 C) 3 D) 5 E) 9

12. Renata digitou um trabalho de 100 páginas numerados de 1 a 100 e oimprimiu. Ao folhear o trabalho, percebeu que sua impressora estava com defeito, pois trocava o zero pelo um e o um pelo zero na numeração das páginas. Depois de consertar a impressora, quantas páginas teve que reimprimir, no mínimo ?A) 18 B) 20 C) 22 D) 30 E) 28

13. Letícia vendeu todos seus CDs de videogames para três amigos, que lhepagaram, respectivamente, R$ 240,00, R$ 180,00 e R$ 320,00. Todos os CDs tinham o mesmo preço. Quantos CDs tinha Letícia no mínimo?A) 20 B) 37 C) 28 D) 21 E) 25

14. 6 cartões com números somente numa das faces são colocados sobre uma mesa conforme a ilustração. Os cartões X e Y estão com a face numerada voltada para baixo. A média aritmética dos números de todos os cartões é 5. A média aritmética dos números do cartão Y e seus três vizinhos é 3. Qual é o número escrito no cartão X ? A) – 4 B) 12 C) 0 D) 15 E) 10

15. Rafael tem da idade de Roberto e é 2 anos mais jovem que Reinaldo. A

idade de Roberto representa da idade de Reinaldo. Em anos, a soma das idades

dos três é:A) 48 B) 72 C) 58 D) 60 E) 34

EUREKA! N°7, 2000

5

Page 6: CONTEÚDO · Web viewA OBM divulga apenas os nomes e pontuações dos alunos premiados; a OBM nunca divulgou nem divulgará comparações entre colégios. Nosso objetivo é estimular

Sociedade Brasileira de Matemática

16. Marcos quer pesar três maças numa balança deDois pratos, mas ele dispõe de apenas um bloco de 200 gramas. Observando o equilíbrio na balança, ele observa que a maçã maior tem o mesmo peso que as outras duas maçãs juntas; o bloco e a maçã menor pesam tanto quanto as outras duas maçãs juntas; a maçã maior junto com a menor pesam tanto quanto bloco. O peso total das três maçãs é:A) 250 g B) 300 g C) 350 g D) 400 g E) 450 g

17. No desenho ao lado estão representadosQuatro triângulos retângulos e um retângulo, bem como suas medidas. Juntando todas essas figuras, podemos construir um quadrado. O lado desse quadrado irá medir:A) 88 cm B) 100 cm C) 60 cm D) 96 cm E) 80 cm

18. Numa certa cidade, foi adotado o seguinte sistema de rodízio de carros: duasvezes por semana, de segunda a sexta, cada carro fica proibido de circular, de acordo com o final de sua placa (algarismo das unidades). O número médio de finais de placa proibidos diferentes para cada dia de proibição é:A) 4 B) 1 C) 3 D) 2 E) indefinido

19. Alexandre, consultando a programação de filmes, decidiu gravar Contato, cuja duração é de 150 minutos. Para gravar numa única fita, ele

começou com velocidade menor (modo EP, que permite gravar 6 horas) e, num dado momento, mudou para a velocidade maior (modo SP, que permite gravar 2 horas), de forma que a fita acabou exatamente no fim do filme. Do início do filme até o momento da mudança do modo de gravação, quantos minutos se passaram?A) 60 B) 30 C) 15 D) 45 E) 105

20. Você sabe que existem 9 números de um algarismo, 90 números de doisalgarismos, 900 números de três algarismos, etc. Considere agora cada número cujo último algarismo à direita representa o número de algarismos desse número. Por exemplo, o número 9 074 é um deles, pois 4 é o número de seus algarismos. Quantos números desse tipo existem ?A) 99 999 999 B) 99 999 992 C) 100 000 000 D) 10 000 000 E) 1 000 000 000

EUREKA! N°7, 2000

6

Page 7: CONTEÚDO · Web viewA OBM divulga apenas os nomes e pontuações dos alunos premiados; a OBM nunca divulgou nem divulgará comparações entre colégios. Nosso objetivo é estimular

Sociedade Brasileira de Matemática

XXI OLIMPÍADA BRASILEIRA DE MATEMÁTICAPrimeira Fase - Nível 2

01. Veja problema 01 do Nível 1.

02. Em um hotel há 100 pessoas. 30 comem porco, 60 comem galinha e 80 comem alface. Qual é o maior número possível de pessoas que não comem nenhum desses dois tipos de carne?A) 10 B) 20 C) 30 D) 40 E) 50

03. Uma folha quadrada foi dobrada duas vezes ao longo de suas diagonaisconforme ilustração abaixo, obtendo-se um triângulo isósceles. Foi feito um corte na folha dobrada, paralelo à base desse triângulo, pelos pontos médios dos outros lados. A área do buraco na folha corresponde a que fração da área da folha original ?

A) B) C) D) E)

04. Veja problema 9 do Nível 1. 05. Veja problema 17 do Nível 1.

06. Contando-se os alunos de uma classe de 4 em 4 sobram 2 e contando-se de 5 em 5 sobra 1. Sabendo-se que 15 alunos são meninas e que nesta classe o número de meninas é maior que o número de meninos, o número de meninos nesta classe é igual a :A) 7 B) 8 C) 9 D) 10 E) 11

07. O quociente de 5050 por 2525 é igual a :A) 2525 B) 1025 C) 10025 D) 225 E) 2 2525

08. Qual o 1999o algarismo após a vírgula na representação decimal de ?

A) 0 B) 1 C) 2 D) 7 E) 8

EUREKA! N°7, 2000

7

Page 8: CONTEÚDO · Web viewA OBM divulga apenas os nomes e pontuações dos alunos premiados; a OBM nunca divulgou nem divulgará comparações entre colégios. Nosso objetivo é estimular

Sociedade Brasileira de Matemática

09. Um retângulo ABCD está dividido em quatro retângulos menores. As áreas de três deles estão na figura abaixo. Qual é a área do retângulo ABCD?

A) 80 B) 84 C) 86 D) 88 E) 91

10. Em um aquário há peixes amarelos e vermelhos: 90% são amarelos e 10% são vermelhos. Uma misteriosa doença matou muitos peixes amarelos, mas nenhum vermelho. Depois que a doença foi controlada verificou-se que no aquário, 75% dos peixes vivos eram amarelos. Aproximadamente, que porcentagem dos peixes amarelos morreram?A) 15% B) 37% C) 50% D) 67% E) 84%

11. Pedro saiu de casa e fez compras em quatro lojas, cada uma num bairro diferente. Em cada uma gastou a metade do que possuía e a seguir, ainda pagou R$ 2,00 de estacionamento. Se no final ainda tinha R$ 8,00, que quantia tinha Pedro ao sair de casa?A) R$ 220,00 B) R$ 204,00 C) R$ 196,00 D) R$ 188,00 E) R$ 180,00

12. Quantos são os possíveis valores inteiros de x para que seja um

número inteiro?A) 5 B) 10 C) 20 D) 30 E) 40

13. A diferença entre a maior raiz e a menor raiz da equação é:

A) 2 B) 3 C) 4 D) 5 E) 6

14. Uma bola de futebol é feita com 32 peças de couro. 12 delas são pentágonos regulares e as outras 20 são hexágonos também regulares. Os lados dos pentágonos são iguais aos dos hexágonos de forma que possam ser costurados. Cada costura une dois lados de duas dessas peças.Quantas são as costuras feitas na fabricação de uma bola de futebol?A) 60 B) 64 C) 90 D) 120 E) 180

EUREKA! N°7, 2000

8

Page 9: CONTEÚDO · Web viewA OBM divulga apenas os nomes e pontuações dos alunos premiados; a OBM nunca divulgou nem divulgará comparações entre colégios. Nosso objetivo é estimular

Sociedade Brasileira de Matemática

15. Hoje, 12/6/1999, Pedro e Maria fazem aniversário. No mesmo dia em 1996, a idade de Pedro era 3/4 da idade de Maria. No mesmo dia em 2002, a idade de Pedro será igual à de Maria quando ele tinha 20 anos. Quantos anos Maria está fazendo hoje?A) 30 B) 31 C) 32 D) 33 E) 34

16. Uma caixa contém 100 bolas de cores distintas. Destas, 30 são vermelhas, 30 são verdes, 30 são azuis e entre as 10 restantes, algumas são brancas e outras são pretas. O menor número de bolas que devemos tirar da caixa, sem lhes ver a cor, para termos a certeza de haver pelo menos 10 bolas da mesma cor é:A) 31 B) 33 C) 35 D) 37 E) 38

17. Quantos são os triângulos que possuem medidas dos seus lados expressas por números inteiros e tais que a medida do maior lado seja igual a 11 ?A) 10 B) 11 C) 12 D) 24 E) 36

18. Os pontos S, T e U são os pontos de tangência do círculo inscrito no triângulo PQR sobre os lados RQ, RP e PQ respectivamente. Sabendo que os comprimentos dos arcos TU, ST e US estão na razão TU : ST : US = 5 : 8 : 11, a razão TPU : SRT : UQS é igual a :A) 7 : 4 : 1 B) 8 : 5 : 2 C) 7 : 3 : 2 D) 11 : 8 : 5 E) 9 : 5 : 1

19. Aos vértices de um cubo são atribuídos os números de 1 a 8 de modo que os conjuntos dos números correspondentes aos vértices das seis faces são {1, 2, 6, 7}, {1, 4, 6, 8}, {1, 2, 5, 8}, {2, 3, 5, 7}, {3, 4, 6, 7} e {3, 4, 5, 8}. O vértice atribuído ao número 6 está mais longe do vértice de número A) 1 B) 3 C) 4 D) 5 E) 7

20. Com os 5 números ímpares entre –5 e 4 e com os 5 números pares entre –5 e 4 são formados 5 pares de números. Se N é a soma dos produtos, obtidos em cada par de números, o valor mínimo possível de N é igual a : A) – 41 B) – 40 C) – 28 D) –10 E) 0

XXI OLIMPÍADA BRASILEIRA DE MATEMÁTICA

EUREKA! N°7, 2000

9

Page 10: CONTEÚDO · Web viewA OBM divulga apenas os nomes e pontuações dos alunos premiados; a OBM nunca divulgou nem divulgará comparações entre colégios. Nosso objetivo é estimular

Sociedade Brasileira de Matemática

Primeira Fase - Nível 3

01. Veja problema 01 do Nível 1.02. Veja problema 02 do Nível 2.

03. Um gafanhoto pula exatamente 1 metro. Ele está em um ponto A de uma reta, só pula sobre ela, e deseja atingir um ponto B dessa mesma reta que está a 5 metros de distância de A com exatamente 9 pulos. De quantas maneiras ele pode fazer isso?A) 16 B) 18 C) 24 D) 36 E) 48

04. Sendo a b e b 0, sabe-se que as raízes da equação sãoexatamente a e b. Então, a – b é igual a:A) 0 B) 1 C) 2 D) 3 E) 4

05. Veja problema 09 do Nível 2.06. Veja problema 14 do Nível 2.

07. A diferença entre a maior raiz e a menor raiz da equação é:

A) 2 B) 3 C) 4 D) 5 E) 6

08. Veja problema 12 do Nível 2.

09. Se 00 < x < 900 e então x está entre:

A) 00 e 300 B) 300 e 450 C) 450 e 600 D) 600 e 750 E) 750 e 900

10. Veja problema 11 do Nível 2.11. Para todo n natural definimos a função f por:

se n é par,

se n é ímpar. O número de soluções da equação é:A) 2 B) 3 C) 4 D) 5 E) 6

12. O número N = 11111 . . . 11 possui 1999 dígitos, todos iguais a 1. O resto da divisão de N por 7 é:A) 1 B) 2 C) 4 D) 5 E) 6

EUREKA! N°7, 2000

10

Page 11: CONTEÚDO · Web viewA OBM divulga apenas os nomes e pontuações dos alunos premiados; a OBM nunca divulgou nem divulgará comparações entre colégios. Nosso objetivo é estimular

Sociedade Brasileira de Matemática

13. Um quadrado ABCD possui lado 40cm. Uma circunferência contém os vértices A e B e é tangente ao lado CD. O raio desta circunferência é:A) 20cm B) 22cm C) 24cm D) 25cm E) 28cm

14. Veja problema 18 do Nível 2.15. Para quantos valores inteiros de x existe um triângulo acutângulo de lados 12, 10 e x?A) 9 B) 10 C) 12 D) 16 E) 18

16. A circunferência abaixo tem raio 1, o arco AB mede 700 e o arco BC mede 400. A área da região limitada pelas cordas AB e AC e pelo arco BC mede:

A) /8 B) /9 C) /10 D) /12 E) /14

17. A reta r contém os pontos (0, 4) e (7, 7). Dos pontos abaixo, qual é o mais próximo da reta r?A) (1999, 858) B) (1999, 859) C) (1999, 860)D) (1999, 861) E) (1999, 862)

18. Quantos são os pares (x, y) de inteiros positivos que satisfazem a equação 2x +3y = 101 ?A) 13 B) 14 C) 15 D) 16 E) 17

19. Quantos números inteiros entre 10 e 1000 possuem seus dígitos em ordem estritamente crescente? (Por exemplo, 47 e 126 são números deste tipo; 52 e 566 não).A) 90 B) 98 C) 112 D) 118 E) 120

20. Veja problema 10 do Nível 2.21. Veja problema 15 do Nível 2.

22. No quadrado ABCD o ponto E é médio de BC e o ponto F do lado CD é tal que o ângulo AEF é reto. Aproximadamente, que porcentagem a área do triângulo AEF representa da área do quadrado?A) 28% B) 31% C) 34% D) 36% E) 39%

EUREKA! N°7, 2000

11

Page 12: CONTEÚDO · Web viewA OBM divulga apenas os nomes e pontuações dos alunos premiados; a OBM nunca divulgou nem divulgará comparações entre colégios. Nosso objetivo é estimular

Sociedade Brasileira de Matemática

23. Dois irmãos herdaram o terreno ABC com a forma de um triângulo retângulo em A, e com o cateto AB de 84m de comprimento. Eles resolveram dividir o terreno em duas partes de mesma área, por um muro MN paralelo a AC como mostra a figura abaixo. Assinale a opção que contém o valor mais aproximado do segmento BM.

A) 55m B) 57m C) 59m D) 61m E) 63m

24. As representações decimais dos números e são escritas lado a lado. O número de algarismos escritos é igual a :A) 1999 B) 2000 C) 2001 D) 3998 E) 3999

25. Veja problema 16 do Nível 2.

GABARITO

Primeiro Nível (5a. e 6a. séries)1) B 6) A 11) E 16) B2) D 7) B 12) E 17) E3) B 8) C 13) B 18) A4) B 9) A 14) E 19) D5) D 10) B 15) C 20) C

Segundo Nível (7a. e 8a. séries)1) B 6) E 11) D 16) E2) D 7) C 12) C 17) E3) E 8) B 13) A 18) A4) A 9) E 14) C 19) D5) E 10) D 15) B 20) B

Terceiro Nível (Ensino Médio)1) B 6) C 11) C 16) B 21) B2) D 7) A 12) A 17) D 22) B3) D 8) C 13) D 18) E 23) C4) D 9) E 14) A 19) E 24) B5) E 10) D 15) A 20) D 25) E

EUREKA! N°7, 2000

12

Page 13: CONTEÚDO · Web viewA OBM divulga apenas os nomes e pontuações dos alunos premiados; a OBM nunca divulgou nem divulgará comparações entre colégios. Nosso objetivo é estimular

Sociedade Brasileira de Matemática

XXI OLIMPÍADA BRASILEIRA DE MATEMÁTICASegunda Fase - Nível 1

PROBLEMA 1Corte 10 algarismos do número 1234512345123451234512345, para que o número restante seja o maior possível.

PROBLEMA 2 Sabe-se que três meses consecutivos de um determinado ano, não bissexto, possuem cada um exatamente quatro domingos. a) Estes meses podem ser janeiro, fevereiro e março?b) Podem ser agosto, setembro e outubro?

PROBLEMA 3Na figura, os triângulos ABC e EGF são equiláteros. O perímetro do triângulo ABC é 132cm e, além disso,

AE = ECBD = DC

EF = FC DG = GEa) Qual o perímetro da área sombreada?b) Que fração da área do triângulo ABC representa a área sombreada?

PROBLEMA 4 Pedro distribuiu 127 moedas de 1 real em sete caixas e colocou em cada uma delas uma etiqueta dizendo o número de moedas da caixa. Essa distribuição foi feita de forma que qualquer quantia de R$1,00 a R$127,00 pudesse ser paga entregando-se apenas caixas fechadas. De que maneira Pedro fez essa distribuição?

PROBLEMA 5 Um edifício muito alto possui 1000 andares, excluindo-se o térreo. Do andar térreo partem 5 elevadores:O elevador A pára em todos os andares. O elevador B pára nos andares múltiplos de 5, isto é, 0, 5, 10, 15, …O elevador C pára nos andares múltiplos de 7, isto é, 0, 7, 14, 21, …O elevador D pára nos andares múltiplos de 17, isto é, 0, 17, 34, 51, …O elevador E pára nos andares múltiplos de 23, isto é, 0, 23, 46, 69, …

EUREKA! N°7, 2000

13

Page 14: CONTEÚDO · Web viewA OBM divulga apenas os nomes e pontuações dos alunos premiados; a OBM nunca divulgou nem divulgará comparações entre colégios. Nosso objetivo é estimular

Sociedade Brasileira de Matemática

a) Mostre que, excetuando-se o andar térreo, não existe nenhum andar onde param os 5 elevadores.b) Determine todos os andares onde param 4 elevadores.

PROBLEMA 6 Encontre o menor tabuleiro quadrado que pode ser ladrilhado usando peças com o seguinte formato:

Obs: Ladrilhado significa completamente coberto, sem superposição de peças, e de modo que nenhum ponto fora do tabuleiro seja coberto por alguma peça.

SOLUÇÕES SEGUNDA FASE - NÍVEL 1

SOLUÇÃO PROBLEMA 1 O maior número restante é 553451234512345. Para ver isto, podemos supor que os cortes são feitos da esquerda para a direita. Se deixarmos de cortar todos os quatro primeiros algarismos, o número que resta começará por 1, 2, 3 ou 4. Logo, menor que o número acima. Feito isto, se deixarmos de cortar a segunda seqüência 1234, o número que resta terá na primeira ou segunda casa, da esquerda para a direita, 1, 2, 3 ou 4. Ainda menor que o número acima. Os dois primeiros 5 devem permanecer, pois retirando-se um deles, completamos 9 retiradas e aí algum algarismo da terceira seqüência 1234 aparecerá na 1a ou na 2a

casa. Finalmente devemos cortar a seqüência 12, que ocupa a 11a e 12a posição.

SOLUÇÃO PROBLEMA 2Se o dia primeiro de janeiro for Segunda-feira, e o ano não for bissexto, então os meses de janeiro, fevereiro e março terão 4 domingos cada.

SOLUÇÃO PROBLEMA 3 (Solução resumida)

a) . b)

SOLUÇÃO PROBLEMA 4Basta distribuir as moedas em 7 caixas contendo respectivamente 1, 2, 4, 8, 16, 32 e 64 moedas. Para outros pagamentos Pedro pode fazer 3 = 1 + 2, 5 = 1 + 4, 6 = 2 + 4, 7 = 1 + 2 + 4. Assim já pode pagar as quantias de 1 a 7 reais com o conteúdo das caixas. Somando-se a parcela de 8 a estas somas chega-se nas somas de 9 até 15. Somando-se a parcela de 16 às 15 somas assim formadas

EUREKA! N°7, 2000

14

Page 15: CONTEÚDO · Web viewA OBM divulga apenas os nomes e pontuações dos alunos premiados; a OBM nunca divulgou nem divulgará comparações entre colégios. Nosso objetivo é estimular

Sociedade Brasileira de Matemática

obtém-se somas de 17 a 31. A estas acrescenta-se a parcela de 32. E finalmente a parcela de 64, obtendo-se assim todas as somas de 1 a 127 = 1+ 2 + 4 + 8 + 16 + 32 + 64.

SOLUÇÃO PROBLEMA 5a) O elevador B pára nos múltiplos de 5.

O elevador C pára nos múltiplos de 7.O elevador D pára nos múltiplos de 17.O elevador E pára nos múltiplos de 23.

Como 5, 7, 17 e 23 são números primos, para que todos parem num mesmo andar, este tem que ser múltiplo de 5 7 17 23 = 13685 e o prédio só tem 1000 andares.

b) Para que num andar parem exatamente quatro elevadores, devem parar A, que pára em todos, e três dos restantes.B, C e D param nos múltiplos de 5 7 17 = 595 B, C e E param nos múltiplos de 5 7 23 = 805B, D e E param nos múltiplos de 5 17 23 = 1955 C, D e E param nos múltiplos de 7 17 23 = 2737 Logo, os andares onde param 4 elevadores são o 595 e o 805.

SOLUÇÃO PROBLEMA 6O menor tabuleiro é do tipo 10 10 coberto com 20 peças, como mostrado, por exemplo, pela figura abaixo, à esquerda.

Com efeito, o número de casas do tabuleiro é um quadrado perfeito múltiplo de 5. Logo é 25, 100, 225 ou ... etc. Mas um tabuleiro 5 5 não pode ser coberto com peças deste tipo, pois ao tentarmos completar uma lateral do tabuleiro, seremos conduzidos a uma das duas figuras à direita, as quais não se deixam completar pelas peças para formar todo o tabuleiro.

XXI OLIMPÍADA BRASILEIRA DE MATEMÁTICA

EUREKA! N°7, 2000

15

Page 16: CONTEÚDO · Web viewA OBM divulga apenas os nomes e pontuações dos alunos premiados; a OBM nunca divulgou nem divulgará comparações entre colégios. Nosso objetivo é estimular

Sociedade Brasileira de Matemática

Segunda Fase - Nível 2

PROBLEMA 1 Três meses consecutivos de um determinado ano, não bissexto, possuem exatamente quatro domingos cada um. Prove que um destes meses é fevereiro.

PROBLEMA 2 Num quadro-negro são escritos três inteiros. Começa-se, então, uma sequência de movimentos onde, em cada passo, apaga-se um deles e escreve-se em seu lugar a soma dos outros dois diminuída de uma unidade. Após vários movimentos, estão escritos no quadro os números 17, 75 e 91. É possível que no início estejam escritos no quadro :

a) 2, 2, 2 ?b) 3, 3, 3 ?

PROBLEMA 3 Seja ABCD um quadrado. Escolhemos pontos M, N, P, Q respectivamente sobre AB, BC, CD e DA, de modo que as circunferências circunscritas aos triângulos MBN e PDQ sejam tangentes exteriormente. Mostre que MN +PQ AC.

PROBLEMA 4 Determine o maior natural n para o qual existe uma reordenação (a, b, c, d) de (3, 6, 9, 12) (isto é, {a, b, c, d} = {3, 6, 9, 12}) tal que o número 3 6 9 12a b c dn seja inteiro. Justifique sua resposta.

PROBLEMA 5

Um professor de matemática passou aos seus alunos a adição onde A, B,

C e D são inteiros positivos, as frações estão simplificadas ao máximo e os denominadores são números primos entre si. Os alunos adicionaram as frações tirando o mínimo múltiplo comum dos denominadores das parcelas e escrevendo este como o denominador do resultado. Mostre que a fração que os alunos encontraram como resultado está simplificada.

PROBLEMA 6 Determine todos os inteiros positivos n para os quais é possível montarmos um retângulo 9 10 usando peças 1 n.

SOLUÇÕES SEGUNDA FASE - NÍVEL 2

EUREKA! N°7, 2000

16

Page 17: CONTEÚDO · Web viewA OBM divulga apenas os nomes e pontuações dos alunos premiados; a OBM nunca divulgou nem divulgará comparações entre colégios. Nosso objetivo é estimular

Sociedade Brasileira de Matemática

SOLUÇÃO PROBLEMA 1 Se nenhum destes meses for fevereiro, o número total de dias não pode ser menor que 91 = 7. 13 e portanto o número total de domingos não poderia ser menor do que 13.

SOLUÇÃO PROBLEMA 2

a) Estão escritos inicialmente 3 números pares. Quando um deles é apagado, é escrito em seu lugar um número ímpar. Após o 1º movimento ficam então dois números pares e um número ímpar. Se apagarmos agora o número ímpar, surgirá em seu lugar outro númro ímpar e se apagarmos um número par aparecerá em seu lugar outro número par. Deste modo, após qualquer número de movimentos restarão dois números pares e um número ímpar e portanto, não é possível termos no final os três números ímpares 17, 75 e 91.

b) Sim, uma possível sequência de movimentos é : 3, 3, 3 5, 3, 3 5, 3, 7 5, 11, 7 17, 11, 7 17 , 11, 27 17, 43, 27 17, 43, 59 17, 75, 59 17, 75, 91.

SOLUÇÃO PROBLEMA 3 A figura abaixo representa a situação, onde X e Y são os pontos médios dos segmentos MN e PQ e Z é o ponto de tangência das circunferências. Então, como

, segue que BX = MX = NX = XZ e DY = QY = YP = YZ. Assim, MN + PQ = BX + XZ + ZY + YD BD = .

SOLUÇÃO PROBLEMA 4 Temos Para (a, b, c, d) dados, o maior n possível é Note que b + 2d é máximo (com b e d elementos distintos de {3, 6, 9, 12}) quando d = 12 e b = 9. Neste caso, b + 2d = 33, e a + b + 2c + d = 21 + a + 2c. Tomando a = 6 e c = 3, temos também a + b + 2c + d = 33, que é obviamente o maior valor possível para n, obtido para (a, b, c, d) = (6, 9, 3, 12).SOLUÇÃO PROBLEMA 5

EUREKA! N°7, 2000

17

Page 18: CONTEÚDO · Web viewA OBM divulga apenas os nomes e pontuações dos alunos premiados; a OBM nunca divulgou nem divulgará comparações entre colégios. Nosso objetivo é estimular

Sociedade Brasileira de Matemática

Como os denominadores das frações são primos entre si, seu MMC é BD e assim,

a fração resultante é . Suponhamos que esta fração não seja irredutível

isto é, que exista algum número primo p que divida o numerador e o denominador desta fração. Como o produto BD é divisível por p, um dos seus termos, digamos B sem perda de generalidade o seja. Entretanto, uma das parcelas da soma AD + CB é divisível por p e como a soma, por hipótese, é divisível por p a parcela AD é também divisível por p. Portanto A ou D é divisível por p. No primeiro caso

temos uma contradição com o fato da fração ser irredutível, no outro casos a

contradição está no fato de que os denominadores das frações iniciais sempre são primos entre si.

SOLUÇÃO PROBLEMA 6 É claro que n deve ser no máximo 10 e dividir 90. Assim, restam para n as possibilidades 1, 2, 3, 5, 6, 9, 10. Fora n = 6, é imediato que n pode assumir qualquer um dos outros valores acima. Começando a tentar montar o retângulo com peças 1 6 a partir de um canto, concluímos prontamente que a tarefa não é possível.

XXI OLIMPÍADA BRASILEIRA DE MATEMÁTICASegunda Fase - Nível 3

EUREKA! N°7, 2000

18

Page 19: CONTEÚDO · Web viewA OBM divulga apenas os nomes e pontuações dos alunos premiados; a OBM nunca divulgou nem divulgará comparações entre colégios. Nosso objetivo é estimular

Sociedade Brasileira de Matemática

PROBLEMA 1 Nos extremos de um diâmetro de um círculo, escreve-se o número 1 (primeiro passo) . A seguir, cada semicírculo é dividido ao meio e em cada um dos seus pontos médios escreve-se a soma dos números que estão nos extremos do semicírculo (segundo passo) . A seguir, cada quarto de círculo é dividido ao meio e em cada um dos seus pontos médios coloca-se a soma dos números que estão nos extremos de cada arco (terceiro passo). Procede-se, assim, sucessivamente: sempre cada arco é dividido ao meio e em seu ponto médio é escrita a soma dos números que estão em seus extremos. Determinar a soma de todos os números escritos após 1999 passos.

PROBLEMA 2 Veja problema 3 do nível 2.PROBLEMA 3 Veja problema 4 do nível 2.

PROBLEMA 4 Determine todos os inteiros positivos n para os quais é possível montarmos um retângulo 9 10 usando peças 1 n.

PROBLEMA 5José tem três pares de óculos, um magenta, um amarelo e um ciano. Todo dia de manhã ele escolhe um ao acaso, tendo apenas o cuidado de nunca usar o mesmo que usou no dia anterior. Se dia primeiro de agosto ele usou o magenta, qual a probabilidade de que dia 31 de agosto ele volte a usar o magenta?

PROBLEMA 6Encontre as soluções inteiras de .

SOLUÇÕES SEGUNDA FASE - NÍVEL 3

SOLUÇÃO PROBLEMA 1

Seja S(n) a soma dos termos em cada passo em um dos semicírculos. Observemos que S(1) = 2, S(2) = 4, e S(3) = 10. Deste modo, nos parece razoável conjecturar que S(n) = 3n 1 +1. Claramente, S(1) = 31 1 + 1. Os novos termos adicionados para formar Ln +1 representam somas de dois termos consecutivos de Ln e cada termo de Ln, excetuando-se o primeiro e o último, aparece em exatamente duas destas somas. Daí, S(n +1) = S(n) + 2(S(n) 1) = 3S(n) – 2 = 3(3n 1 + 1) – 2 = 3(n + 1) 1 + 1. Levando em consideração o outro semicírculo, temos que a soma após os 1999 passos é igual a 2.(31999 1 + 1) 2 = 2. 31998

EUREKA! N°7, 2000

19

Page 20: CONTEÚDO · Web viewA OBM divulga apenas os nomes e pontuações dos alunos premiados; a OBM nunca divulgou nem divulgará comparações entre colégios. Nosso objetivo é estimular

Sociedade Brasileira de Matemática

SOLUÇÃO PROBLEMA 2 Veja solução do problema 3 do nível 2.SOLUÇÃO PROBLEMA 3 Veja solução do problema 4 do nível 2.

SOLUÇÃO PROBLEMA 4É claro que n deve ser no máximo 10 e dividir 90. Assim, restam para n as possibilidades 1, 2, 3, 5, 6, 9, 10. Fora n = 6, é imediato que n pode assumir qualquer um dos outros valores acima. Começando a tentar montar o retângulo com peças 1 6 a partir de um canto, concluímos prontamente que a tarefa não é possível.

SOLUÇÃO PROBLEMA 5Sejam mn , an e cn as probabilidades de que no dia n ele use óculos magenta,

amarelo e ciano, respectivamente. Temos m1 = 1, a1 = c1 = 0 e ,

, e Como an + cn + mn = 1, temos

Assim, , e em 31 de agosto a probabilidade de que ele volte a

usar o magenta é

SOLUÇÃO PROBLEMA 6Temos . Suponhamos x > y. Assim, os possíveis valores de a = x – y são 1, 3, 9, 27, 37, 3 37, 9 37, 27 37 e cada valor permite fazer y = x – a e precisamos apenas verificar se as raízes de

são inteiras. Na verdade, alguns destes valores são

obviamente inapropriados: , donde os valores 1 e 37 podem ser descartados. Por outro lado, se temos

donde podemos descartar a 27. Os dois valores restantes, 3 e 9, são de fato possíveis e dão as quatro soluções:

XXI OLIMPÍADA BRASILEIRA DE MATEMÁTICATerceira Fase - Nível 1

PROBLEMA 1 Diga como dividir um cubo em 1999 cubinhos. A figura mostra uma maneira de dividir um cubo em 15 cubinhos.

EUREKA! N°7, 2000

20

Page 21: CONTEÚDO · Web viewA OBM divulga apenas os nomes e pontuações dos alunos premiados; a OBM nunca divulgou nem divulgará comparações entre colégios. Nosso objetivo é estimular

Sociedade Brasileira de Matemática

PROBLEMA 2Emanuela, Marta e Isabel são três nadadoras que gostam de competir e por isso resolveram organizar um desafio de natação entre elas. Ficou combinado o total de pontos para o primeiro, o segundo e o terceiro lugares em cada prova. A pontuação para primeiro lugar é maior que a para o segundo e esta é maior que a pontuação para o terceiro. As pontuações são números inteiros positivos. O desafio consistiu de várias provas e ao final observou-se que Emanuela fez 20 pontos, Marta 9 pontos e Isabel 10. A primeira prova foi vencida por Isabel.

(a) Quantas provas foram disputadas?(b) Determine o total de pontos para o primeiro, segundo e terceiro lugares.

PROBLEMA 3Um reino é formado por dez cidades. Um cidadão muito chato foi exilado da cidade A para cidade B, que é a cidade do reino mais longe de A. Após um tempo, ele foi expulso da cidade B para a cidade C do reino mais longe de B. Sabe-se que a cidade C não é a mesma cidade A. Se ele continuar sendo exilado dessa maneira, é possível que ele retorne à cidade A?

Nota: as distâncias entre as cidades são todas diferentes.

PROBLEMA 4 Adriano, Bruno e Carlos disputaram uma série de partidas de tênis de mesa. Cada vez que um jogador perdia, era substituído pelo que estava a esperar. A primeira partida foi disputada por Adriano e Bruno. Sabe-se que Adriano venceu 12 partidas e Bruno 21. Quantas vezes Adriano e Bruno se enfrentaram?SOLUÇÕES TERCEIRA FASE - NÍVEL 1

PROBLEMA 1 SOLUÇÃO DE MARIANA DE MORAES SILVEIRA (Belo Horizonte - MG) O cubo deve ser dividido em 1000 cubinhos, ou seja 10 10 10, depois, deve-se pegar um deles e dividí-lo novamente em 1000 cubinhos para que obtenhamos

EUREKA! N°7, 2000

21

Page 22: CONTEÚDO · Web viewA OBM divulga apenas os nomes e pontuações dos alunos premiados; a OBM nunca divulgou nem divulgará comparações entre colégios. Nosso objetivo é estimular

Sociedade Brasileira de Matemática

1999 cubinhos. Assim teremos 1000 – 1 (que será dividido) + 1000 = 1999 cubinhos.

PROBLEMA 2SOLUÇÃO DE DIOGO DOS SANTOS SUYAMA (Belo Horizonte - MG)a) Foram disputadas 3 provas. Como 20 + 10 + 9 = 39, o número de pontos

distribuidos por prova só poderia ser 3 ou 13, pois estes são os únicos divisores de 39, a não ser o mesmo e o 1. Em consequências, o número de provas também será um desses números. Porém, se forem disputadas 13 provas, só há uma maneira de se distribuir os pontos: 2 para o primeiro, 1 para o segundo e 0 para o terceiro. Entretanto, 0 não é positivo, sendo assim descartada essa hipótese.

b) Já sabendo que são 3 provas, é impossível que a vencedora ganhe menos que 8 pontos, pois assim, Emanuela só conseguiria os 20 pontos fazendo 7, 7 e 6 pontos em cada prova. Para isso, seria preciso que a vencedora fizesse 7 pontos, a segunda colocada 6 e a última 0, mas como vimos, 0 não é positivo. É impossível, também que a vencedora faça mais de 10 pontos, pois não seria possível que a segunda fizesse mais pontos que a última, ou que esta não fizesse 0 pontos. Então, as únicas possibilidades são: 1a. 10, 2a. 2, 3a. 1; 1a. 9, 2a. 3, 3a. 1; 1a. 8, 2a. 4, 3a. 1; e 1a. 8, 2a. 3, 3a. 2. A primeira opção é incorreta, pois Isabel, que venceu uma das provas, não poderia ter feito pontos nas outras. A segunda opção também não é correta, pois Isabel teria que marcar apenas um ponto em duas provas. A última opção é incorreta também, pois Isabel teria que marcar 2 pontos em duas provas. Terceira opção: 1a. 8, 2a. 4, 3a. 1 é a correta. Veja o quadro abaixo:

1a. Prova 2a. Prova 3a. Prova TotalEmanuela 4 8 8 20

Marta 1 4 4 9Isabel 8 1 1 10

PROBLEMA 3 Veja solução do problema 2 do nível 2.PROBLEMA 4 Veja solução do problema 3 do nível 2

XXI OLIMPÍADA BRASILEIRA DE MATEMÁTICATerceira Fase - Nível 2

PROBLEMA 1 Seja ABCDE um pentágono regular tal que a estrela ACEBD tem área 1. Sejam P interseção entre AC e BE e Q a interseção entre BD e CE. Determine a área de APQD.

EUREKA! N°7, 2000

22

Page 23: CONTEÚDO · Web viewA OBM divulga apenas os nomes e pontuações dos alunos premiados; a OBM nunca divulgou nem divulgará comparações entre colégios. Nosso objetivo é estimular

Sociedade Brasileira de Matemática

PROBLEMA 2Um reino é formado por dez cidades. Um cidadão muito chato foi exilado da cidade A para a cidade B, que é a cidade do reino mais longe de A. Após um tempo, ele foi expulso da cidade B para a cidade C do reino mais longe de B. Sabe-se que a cidade C não é a mesma cidade A. Se ele continuar sendo exilado dessa maneira, é possível que ele retorne à cidade A?

Nota: as distâncias entre as cidades são todas diferentes.

PROBLEMA 3Adriano, Bruno e Carlos disputaram uma série de partidas de tênis de mesa. Cada vez que um jogador perdia, era substituído pelo que estava a esperar. A primeira partida foi disputada por Adriano e Bruno. Sabe-se que Adriano venceu 12 partidas e Bruno 21. Quantas vezes Adriano e Bruno se enfrentaram?

PROBLEMA 4 Prove que há pelo menos um algarismo diferente de zero entre a 1.000.000a. e a 3.000.000a. casa decimal de após a vírgula.

SOLUÇÕES TERCEIRA FASE - NÍVEL 2

PROBLEMA 1 Veja solução do problema 1 do nível 3.

PROBLEMA 2SOLUÇÃO DE EINSTEIN DO NASCIMENTO JÚNIOR ( Fortaleza - CE )Há dez cidades A, B, C, D, E, F, G, H, I, J.Um chato da cidade A foi exilado para a cidade mais longe de A, a cidade B.

EUREKA! N°7, 2000

23

Page 24: CONTEÚDO · Web viewA OBM divulga apenas os nomes e pontuações dos alunos premiados; a OBM nunca divulgou nem divulgará comparações entre colégios. Nosso objetivo é estimular

Sociedade Brasileira de Matemática

Como B é a cidade mais longe de A, pode-se dizer que se tomarmos A como sendo o centro de uma circunferência de raio AB, todas as cidades estarão dentro dos limites da circunferência, exceto a cidade B que estará em cima dela.

Como as distâncias entre as cidades não são iguais e o chato foi exilado para a cidade C que é a mais longe de B então BC > AB.Da cidade C ele será exilado para a cidade D que é a mais longe de C e assim sucessivamente até chegar na cidade J onde teremos a seguinte verdade:AB < BC < CD < . . . < HI < IJ.Ao chegar nesse ponto vemos que A com certeza não é a cidade mais longe de J poisAB = raioAJ < raioAJ < ABAB < IJAJ < IJLogo ele irá para uma cidade diferente de A, e nunca retornará à cidade A.

PROBLEMA 3SOLUÇÃO DE FÁBIO DIAS MOREIRA (Rio de Janeiro - RJ) Quando começa a série, já ocorre um encontro entre Adriano (A) e Bruno (B). Vamos chamar de VA, VB e VC o número de vitórias de Adriano, Bruno e Carlos, respectivamente. Então ao final da série VA + VB = 33 e depois do 1o. jogo VA + VB

= 1. Suponhamos que o segundo jogo seja x C. Chamemos de E o número de jogos A B.Então no 2o. jogo E = 1. Enquanto C ganhar, VA + VB e E permanecem constantes. Quando C perder, VA + VB aumenta uma unidade. O próximo jogo será A B, aumentando VA + VB e E em uma unidade. Após este jogo, o próximo será x C. Ou seja, para que E aumente uma unidade, VA + VB aumenta duas, e o aumento de um em E. Como no 2o. jogo E = 1 e falta que VA + VB aumente 32 unidades, ocorrem 1 + 16 = 17 jogos A B.

EUREKA! N°7, 2000

24

Page 25: CONTEÚDO · Web viewA OBM divulga apenas os nomes e pontuações dos alunos premiados; a OBM nunca divulgou nem divulgará comparações entre colégios. Nosso objetivo é estimular

Sociedade Brasileira de Matemática

PROBLEMA 4SOLUÇÃO DE HENRIQUE CHOCIAY (Pinhais - PR)Para começar a desenvolver , utilizei o processo de extração que não utiliza tentativas (processo prático por aproximação).

Deste lado, o número de casas sempre aumenta em 1 casa, nunca mais. (mesmo se houvesse um caso de 99999 9 = 899991 (só aumenta 1 casa) (entre 1.000.000 e 3.000.000)

Quando estivermos no número 1.000.000 de casas no multiplicador, teremos 999.999 casas decimais. Supondo que haja só 1 casa no resto nesta situação, depois de 1.000.000 de operações, teremos 1.999.999 casas decimais (1 milhão de zeros), 2.000.000 no multiplicador e 2.000.001 no resto, podendo obter número diferente de zero.Em geral o fato de, não podendo haver divisão, com o aumento das casas divisoras em 1 e do resto em 2 e as casas decimais serem menores que as divisoras em 1 torna impossível a obtenção desta seqüência de zeros entre as casas de 1.000.000 e 3.000.000.

XXI OLIMPÍADA BRASILEIRA DE MATEMÁTICATerceira Fase - Nível 3

PRIMEIRO DIA

PROBLEMA 1 Veja problema 1 do nível 2.PROBLEMA 2 Veja problema 4 do nível 2.

PROBLEMA 3

EUREKA! N°7, 2000

25

Page 26: CONTEÚDO · Web viewA OBM divulga apenas os nomes e pontuações dos alunos premiados; a OBM nunca divulgou nem divulgará comparações entre colégios. Nosso objetivo é estimular

Sociedade Brasileira de Matemática

Temos um tabuleiro quadrado 10 10.Desejamos colocar n peças em casas do tabuleiro de tal forma que não existam 4 peças formando em retângulo de lados paralelos aos lados do tabuleiro.Determine o maior valor de n para o qual é possível fazer esta construção.

SEGUNDO DIA

PROBLEMA 4O planeta Zork é esférico e tem várias cidades. Dada qualquer cidade existe uma cidade antípoda (i.e., simétrica em relação ao centro do planeta).Existem em Zork estradas ligando pares de cidades. Se existe uma estrada ligando as cidades P e Q então também existe uma estrada ligando as cidades P' e Q', onde P' é a antípoda de P e Q' é a antípoda de Q. Além disso, estradas não se cruzam e dadas duas cidades P e Q sempre é possível viajar de P a Q usando alguma seqüência de estradas.O preço da Kriptonita em Urghs (a moeda planetária) em duas cidades ligadas por uma estrada difere por no máximo 100 Urghs. Prove que existem duas cidades antípodas onde o preço da Kriptonita difere por no máximo 100 Urghs.

PROBLEMA 5Em Tumbólia existem n times de futebol .Deseja-se organizar um campeonato em que cada time joga exatamente uma vez com cada um dos outros. Todos os jogos ocorrem aos domingos, e um time não pode jogar mais de uma vez no mesmo dia. Determine o menor inteiro positivo m para o qual é possível realizar um tal campeonato em m domingos.

PROBLEMA 6Dado triângulo ABC mostre como construir com régua e compasso um triângulo A’B’C’de área mínima com tal que

EUREKA! N°7, 2000

26

Page 27: CONTEÚDO · Web viewA OBM divulga apenas os nomes e pontuações dos alunos premiados; a OBM nunca divulgou nem divulgará comparações entre colégios. Nosso objetivo é estimular

Sociedade Brasileira de Matemática

SOLUÇÕES TERCEIRA FASE - NÍVEL 3

PROBLEMA 1 SOLUÇÃO DE HUGO PINTO IWATA (São José do Rio Preto - SP)

Como o pentágono e a estrela são regulares, o quadrilátero APQD é um trapézio.A área do trapézio APQD é igual à área do triângulo APD somada à do triângulo PQD. Como BDRP também é um trapézio, então a área de PQD é igual à de RQD. Como a estrela é regular, a área de RQD é igual à de ERS, então, a área de PQD é igual à de ERS. Assim a área do trapézio APQD é igual à soma das áreas dos triângulos APD e ERS, que é igual à figura APDRES, que é exatamente metade da estrela toda. Resposta: A área de APQD é 0,5.

PROBLEMA 2

EUREKA! N°7, 2000

27

Page 28: CONTEÚDO · Web viewA OBM divulga apenas os nomes e pontuações dos alunos premiados; a OBM nunca divulgou nem divulgará comparações entre colégios. Nosso objetivo é estimular

Sociedade Brasileira de Matemática

SOLUÇÃO DE HUMBERTO SILVA NAVES (Goiânia - GO)

Suponhamos, por absurdo, que todos os algarismos das casas decimais entre a 1.000.000a. casa decimal e a 3.000.000a. casa decimal de fossem zero, então:

(onde

mas como , (pois se não fosse teríamos um absurdo, pois é irracional!) então:

mas como temos (pela definição de ):

logo:

um absurdo, pois não existe

nenhum inteiro maior que 0 e menor que 1, disto concluímos que há um algarismo diferente de 0 nestas casas decimais. (Poderíamos ter uma aproximação melhor pois 2K é bem menor que ).Obs: denota a função do "maior inteiro": é o único inteiro tal que

PROBLEMA 3 SOLUÇÃO DA BANCA O problema é equivalente a encontrar subconjuntos A1, A2, …, A10 do conjunto {1, 2, 3, …, 10} cuja soma do número de elementos seja a maior possível tais que a interseção de dois quaisquer deles tenha no máximo um elemento (Ai é o conjunto das posições das peças na i-ésima linha do tabuleiro). Se Ai tem ki

elementos então há subconjuntos de 2 elementos não pode

pertencer a dois dos conjuntos Ai, e há no total subconjuntos de 2 elementos de

EUREKA! N°7, 2000

28

Page 29: CONTEÚDO · Web viewA OBM divulga apenas os nomes e pontuações dos alunos premiados; a OBM nunca divulgou nem divulgará comparações entre colégios. Nosso objetivo é estimular

Sociedade Brasileira de Matemática

{1, 2,…,10}. Assim, devemos ter

Por outro lado, se existem i, j com kj > ki + 1, temos

Assim para minimizar mantendo fixo devemos ter

para todo i, j. Se observamos que concluímos que se

então valendo a igualdade se e só se

5 dos ki são iguais a 4 e os outros 5 iguais a 3. Para que a contrução seja possível nesse caso precisamos de que cada par de elementos apareça em exatamente um dos conjuntos Ai . Nesse caso, cada elemento de {1, 2, 3…, 10} deve aparecer em 3 conjuntos com 4 elementos ou em um conjunto com 4 elementos e 3 conjuntos com 3 elementos (pois cada um dos outros 9 elementos aparece exatamente uma vez junto com ele). Como haveria 5 conjuntos com 4 elementos, o número médio de conjuntos com 4 elementos aos quais cada elemento pertence é 2, donde há elementos que pertencem a 3 conjuntos com 4 elementos (pois um elemento não pode pertencer a exatamente 2 conjuntos com 4 elemetos). Assim, podemos supor sem perda de generalidade que A1 = {1, 2, 3, 4}, A2 = {1, 5, 6, 7} e A3 = { 1, 8, 9, 10}, mais então qualquer outro conjunto de 4 elementos deve estar contido em {2, 3, …, 10}, e portanto deve intersectar um dos conjunto A1, A2, A3, A4, em pelo

menos 2 elementos. Portanto, não é possível que seja igual a 35. Por outro

lado é possível construir exemplos com como abaixo:

A1 = {1, 2, 3, 4}, A2 = {1, 5, 6, 7}, A3 = { 2, 5, 8, 9}, A4 = {3, 6, 8, 10}, A5 = {1, 9, 10}, A6 = { 2, 7, 10}, A7 = {3, 7, 9}, A8 = {4, 5, 10}, A9 = { 4, 6, 9} e A10 = {4, 7, 8}.

EUREKA! N°7, 2000

29

Page 30: CONTEÚDO · Web viewA OBM divulga apenas os nomes e pontuações dos alunos premiados; a OBM nunca divulgou nem divulgará comparações entre colégios. Nosso objetivo é estimular

Sociedade Brasileira de Matemática

PROBLEMA 4 SOLUÇÃO DE GILBERTO SANTOS DO NASCIMENTO (São Paulo - SP)Seja o antípoda de

Vamos ligar e vice-versa, formando uma linha fechada. Abaixo é o antípoda de para todo j.

Agora, supondo que a diferença da Kriptonita de para seja maior que 100. Então, vamos supor que (C2 – C1) + (C3 – C2) +…+ (C1' – Cn) > 100. Como ao percorrer o caminho, temos de ter uma diferença zero ao chegarmos em C1

novamente, somando (C2' – C1') + (C3' – C2') +…+ (C1 – Cn') < – 100.Agora, supondo que o Superman trace uma linha de C1 a C1' (esta linha não poderia ser uma estrada, pois | C1' – C1| > 100) a soma das diferenças na parte de cima da linha deve ser maior que 100 e embaixo menor que –100.

Agora, supondo que esta linha percorra a figura, ligando todas as cidades antípodas, na parte de cima, a soma deve continuar sendo maior que 100 e embaixo menor que 100. Em p.c. (parte de cima), a soma não pode passar bruscamente de > 100 para < –100, pois são somadas apenas duas diferenças de cada vez (menores que 200 no total!). Assim, para que

EUREKA! N°7, 2000

30

Page 31: CONTEÚDO · Web viewA OBM divulga apenas os nomes e pontuações dos alunos premiados; a OBM nunca divulgou nem divulgará comparações entre colégios. Nosso objetivo é estimular

Sociedade Brasileira de Matemática

p.c. fique negativo < –100 e p.b. fique positivo > 100, teríamos de ter duas cidades antípodas com diferença > 100 em módulo.

Continuando o percurso, ao chegarmos em C1', teremos de ligá-lo a C1. No entanto, p.c. estará em baixo e a soma das diferenças na direção de C1' para C1

terá de ser positivo > 100. Mas essa soma era negativa e < –100 quando começamos () Contradição.O mesmo ocorre analogamente com p.b. Logo, em algum par da cidades (uma cidade e sua antípoda), a diferença do preço da Kriptonita deverá ser menor ou igual a 100.Viva o Superman!.

PROBLEMA 4 SOLUÇÃO DE HUMBERTO SILVA NAVES (Goiânia - GO) Suponhamos, por absurdo, que os preços diferem por mais de 100 Urghs em todas as cidades antípodas, então: | x0 – y0| > 100 M0 – m0 > 100 (onde xn e yn são antípodas e representam o preço da Kriptonita).| x1 – y1| > 100 M1 – m1 > 100

| xn – yn| > 100 Mn – mn > 100 (Onde Mn = máx (xn, yn) e mn = min (xn, yn))

Como sabemos que existe um caminho de estradas que leva de M0 até m0, então deve existir uma estrada que liga (para certo i, j N; i, j n) Mi mj.Como existe uma estrada ligando Mi mj, também existe uma estrada ligando mj Mi (antípodas). Pode acontecer i = j, caso em que se conclui facilmente que Mi – mi > 100, um absurdo pois mi e Mi são "vizinhas", logo o preço da Kriptonita difere por no máximo 100 Urghs.

EUREKA! N°7, 2000

31

Page 32: CONTEÚDO · Web viewA OBM divulga apenas os nomes e pontuações dos alunos premiados; a OBM nunca divulgou nem divulgará comparações entre colégios. Nosso objetivo é estimular

Sociedade Brasileira de Matemática

Se i j, então:| Mj – mi | 100 (são "vizinhas")| Mi – mj | 100, mas como Mi – mi > 100 e Mj – mj > 100, então: Mi + Mj – mi – mj > 200Mi – mj + Mj – mi > 200 | Mi – mj + Mj – mi| > 200 | Mi – mj| + | Mj – mi| > 200 200 | Mi – mj| + | Mj – mi| > 200, um absurdo, logo existem cidades antípodas cujo preço difere no máximo em 100 Urghs.

PROBLEMA 5 SOLUÇÃO DE FABRÍCIO SIQUEIRA BENEVIDES (Fortaleza - CE)

Façamos 2 casos, n par e n ímpar.

i) n par.

Cada time tem que jogar com cada um dos outros. Se os times são: T1, T2, …, Tn; temos que um time Ti tem que jogar (n – 1) vezes e para isso precisará de pelo menos (n – 1) domingos. (pois só pode jogar 1 vez por domingo). Mostraremos que é possível realizar o campeonato em (n – 1) domingos. Para isso basta que o jogo entre Ti e Tj (i j) ocorra no seguinte domingo.

1) dij i + j (mod n – 1), 1 dij n – 1 para i n, j n 2) din 2i (mod n – 1), 1 din n – 1 para todo i n, j n

(se um dos times for Tn).

Podemos observar isso numa tabela que indique o dia entre Ti e Tj

Exemplo: para n = 6

dij T1 T2 T3 T4 T5 T6

T1 3 4 5 1 2T2 3 5 1 2 4T3 4 5 2 3 1T4 5 1 2 4 3T5 1 2 3 4 5T6 2 3 4 5 1

O campeonato organizado assim satisfaz o problema pois: é fácil ver que um time i joga com cada um dos outros times (no domingo dij, j i). E cada time só joga

EUREKA! N°7, 2000

32

Page 33: CONTEÚDO · Web viewA OBM divulga apenas os nomes e pontuações dos alunos premiados; a OBM nunca divulgou nem divulgará comparações entre colégios. Nosso objetivo é estimular

Sociedade Brasileira de Matemática

uma vez num mesmo dia, caso contrário teríamos: um time Ti que joga contra Tj e Tk no mesmo domingo, ou seja dij = dki

1) Se i = n: djn = dkn 2j 2k (mod n – 1) como (2, n – 1) = 1 teriamos j k (mod n – 1), {j, k} {1, 2, …, n – 1} j = k, uma contradição.

2) Se i n.

2.1) j e k n : dik = dij i + k i + j (mod n – 1) j k (mod n – 1) e k = j. uma contradição.

2.2) j = n, k n, sem perda de generalidade: din = dik i + i i + k (mod n – 1) i k (mod n – 1), {i, j}{1, 2, …, n – 1} i = j, uma contradição.

Agora se n for ímpar, como cada time tem que jogar com todos os outros seria necessário pelo menos (n – 1) domingos.Só que (n – 1) domingos não são suficientes pois em cada dia há um time que fica sem jogar. Assim, se no primeiro dia Ti foi o time que não jogou, ele ainda precisará de mais ( n – 1) domingos para jogar contra os outros. De modo que são necessários pelo menos n domingos.Para ver que n domingos são suficientes, basta que o campeonato se organize assim: Sejam T1, T2, …, Tn os times. Criamos um time virtual chamado Tn + 1 onde jogar contra Tn + 1 um certo dia, significa não jogar naquele dia.Temos então n + 1 = x times, organizamos então como no caso anterior o campeonato. Como x é par isso pode ser feito em x – 1 = n dias.Obs: O exemplo para (2k – 1) times é obtido do de (2k) times esquecendo-se um dos times.

Resposta: Se n é par m = n – 1. Se n é ímpar m = n.

PROBLEMA 6SOLUÇÃO DA BANCA

EUREKA! N°7, 2000

33

Page 34: CONTEÚDO · Web viewA OBM divulga apenas os nomes e pontuações dos alunos premiados; a OBM nunca divulgou nem divulgará comparações entre colégios. Nosso objetivo é estimular

Sociedade Brasileira de Matemática

Sejam A, B, C os ângulos internos do triângulo ABC, sejam A', B', C' os ângulos internos do triângulo A'B'C' e consideremos A' = A e C' = C.Seja D o ponto de interseção das circunferências circuscritas aos triângulos AA'C' e CC'B'. Nos quadriláteros inscritíveis AA'DC' e CC'DB' temos A'DC' = π – A e C'DB' = π – C. Logo, A'DB' = 2π – (π – A) – (π – C) = π – B, e portanto, a circunferência circunscrita ao triângulo BB'A' passa por D.

No quadrilátero inscritível AA'DC', DAA' = DC'A' = e DA'C' = DAC' = a. Como A = A' concluimos que DA'B' = . Logo, no quadrilátero inscritível BB'DA' temos que DBB' = . No quadrilátero inscritível CC'DB' temos que DCB' = DC'B' = c, e como C = C' concluímos que DCC' = . O ponto D está então associado ao triângulo ABC pela propriedade:

DAB = DBC = CDA

e portanto não depende da posição de A', B' e C'. O ponto D é fixo e sua construção será mostrada no final da solução.

Como os ângulos A'DB', B'DC' e C'DA' são constantes, a menor área possível do triângulo A'B'C' é obtida quando os segmentos DA', DB' e DC' forem os menores possíveis. Logo, DA', DB' e DC' são respectivamente perpendiculares aos lados AB, BC e CA.

Construção do ponto D

EUREKA! N°7, 2000

34

Page 35: CONTEÚDO · Web viewA OBM divulga apenas os nomes e pontuações dos alunos premiados; a OBM nunca divulgou nem divulgará comparações entre colégios. Nosso objetivo é estimular

Sociedade Brasileira de Matemática

Seja E a interseção da mediatriz de AB com a perpendicular a BC traçada por B. A circunferência de centro E e raio EA = EB é tangente em B à reta BC. Logo, para qualquer ponto X do menor arco AB tem-se que XAB = XBC.

Seja F a interseção da mediatriz de BC com a perpendicular a CA traçada por C. A circunferência de centro F e raio FB = FC é tangente em C à reta CA. Logo, para qualquer ponto X do menor arco BC tem-se que XBC = XCA.

O ponto D, interseção desses dois arcos é tal que DAB = DBC = DCA.(Note que qualquer ponto D com esta propriedade deve pertencer a cada um dos lugares geométricos descritos acima, o que nos dá a unicidade).

XXI OLIMPÍADA BRASILEIRA DE MATEMÁTICAResultado - Primeiro Nível (5a. e 6a. séries)

NOME CIDADE – ESTADO PRÊMIOHenry Wei Cheng Hsu São Paulo – SP OuroDiogo dos Santos Suyama Belo Horizonte – MG OuroSergio Santos do Nascimento São Paulo – SP OuroGustavo Eufrásio Farias Fortaleza – CE OuroLuciano Lacerda Silveira Campo Grande – MS PrataEmanuel Augusto Varussa Padovan Rio Claro – SP PrataFabrício Henrique de Faria São Paulo – SP PrataThiago Jorge Marinho Vieira Fortaleza – CE PrataPaulo Roberto Sampaio Santiago Salvador – BA PrataMariana de Moraes Silveira Belo Horizonte – MG PrataGabriel Vieira Lana Belo Horizonte – MG BronzeJoão Cláudio Telles Vianna Rio de Janeiro – RJ BronzeRafael Daigo Hirama Campinas – SP BronzeAna Cláudia de Franco Suzuki São Paulo – SP BronzeLuiza de Almeida Aoki S. J. dos Campos – SP BronzeBruno Leonardo Schneider São José – SC BronzePaulo Rebello Bortolini Jundiaí – SP Bronze

EUREKA! N°7, 2000

35

Page 36: CONTEÚDO · Web viewA OBM divulga apenas os nomes e pontuações dos alunos premiados; a OBM nunca divulgou nem divulgará comparações entre colégios. Nosso objetivo é estimular

Sociedade Brasileira de Matemática

Victor Mesquita Barbosa Fortaleza – CE BronzeThiago Augusto Caldas Bello Salvador – BA BronzeSinuhe Djin Maschio Shin São Paulo – SP BronzeRaul Máximo Alexandrino Nogueira Fortaleza – CE BronzeBruno Fiorio Fortaleza – CE BronzePedro H. Milet Pinheiro Pereira Rio de Janeiro – RJ BronzeBernardo Melo Sobreira Fortaleza – CE BronzeMário Luiz Aranha da Silva Salvador – BA BronzeConrado F. Paulo da Costa Rio de Janeiro – RJ BronzeRodrigo Aguiar Pinheiro Fortaleza – CE BronzeDaniel Medeiros de Albuqerque Fortaleza – CE BronzeGabriela Duarte Costa Constantino Timóteo – MG BronzeTiago Porto Barbosa Fortaleza – CE Menção HonrosaVitor Henrique Gonçalves São Carlos – SP Menção HonrosaGabriel Tomé de Lima Mogi das Cruzes – SP Menção HonrosaGustavo Pinheiro Melo Fortaleza – CE Menção HonrosaTúlio Ivo Cordeiro Fulálio Campina Grande – PB Menção HonrosaLeonardo Lucas Rentz Maceió – AL Menção HonrosaDaniela Satie Kondo São Paulo – SP Menção HonrosaRafael Santos Correia de Araujo Salvador – BA Menção HonrosaFelipe Paupitz Schlichting Florianópolis – SC Menção HonrosaÁlinson Santos Xavier Fortaleza – CE Menção HonrosaAntonia Taline de Souza Mendonça Fortaleza – CE Menção HonrosaGustavo Hübner Campina Grande – PB Menção HonrosaLeonardo Deeke Boguszewski Curitiba – PR Menção HonrosaPaola Valente Giorgini Rio de Janeiro – RJ Menção HonrosaRoberta Pieroni Visconti São Paulo – SP Menção HonrosaAlan Hideki Uchida São Paulo – SP Menção HonrosaCincinato Furtado Leite Neto Fortaleza – CE Menção HonrosaMarcus Edson Barreto Brito Fortaleza – CE Menção HonrosaThiago de Sá Jorge Curitiba – PR Menção HonrosaVento Inte Nunes Vieira Curitiba – PR Menção Honrosa

Resultado - Segundo Nível (7a. e 8a. séries)

NOME CIDADE – ESTADO PRÊMIOHenrique Chociay Pinhais – PR OuroDavi Máximo Alexandrino Nogueira Fortaleza – CE OuroMaurício Massao Soares Matsumoto São Paulo – SP OuroFábio Dias Moreira Rio de Janeiro – RJ OuroEduardo Kunio Kuroda Abe São Paulo – SP OuroLarissa de Lima Fortaleza – CE PrataEinstein do Nascimento Júnior Fortaleza – CE PrataDiego Cortez Gutierrez S. J. dos Campos – SP PrataBernardo Freitas Paulo da Costa Rio de Janeiro – RJ PrataBruno Koga Fortaleza – CE PrataRafael Tajra Fonteles Teresina – PI PrataAndré Luis Hirschfeld Danila São Paulo – SP PrataRodrigo Barbosa dos Santos Stein Vitória – ES PrataDaniel Pessôa Martins Cunha Fortaleza – CE BronzeJaquelyne Gurgel Penaforte Fortaleza – CE BronzeThiago Braga Cavalcante Fortaleza – CE BronzeHenrique Cortada Barbieri São Paulo – SP BronzeVinícius Piovesan de Toledo Jundiaí – SP BronzeLucas Gabriel Maltoni Romano Jundiaí – SP BronzeDanilo Vieira Castejon Goiânia – GO BronzeThiago da Silva Sobral Fortaleza – CE BronzeGuilherme Oliveira Campos Bauru – SP BronzeEduardo Barbosa Araújo Fortaleza – CE BronzeTatyana Zabanova Campinas – SP BronzeRafael Montorfano Franco Maringá – PR BronzeOtacílio Torres Vilas Boas Salvador – BA BronzeHenrique Fernandes Macedo Juiz de Fora – MG Menção HonrosaVinicius de Aguiar Furvie São Paulo – SP Menção Honrosa

EUREKA! N°7, 2000

36

Page 37: CONTEÚDO · Web viewA OBM divulga apenas os nomes e pontuações dos alunos premiados; a OBM nunca divulgou nem divulgará comparações entre colégios. Nosso objetivo é estimular

Sociedade Brasileira de Matemática

Renato R. Sinohara da S. Souza S. J. dos Campos – SP Menção HonrosaDaniel Teixeira Brasília – DF Menção HonrosaJefferson Ho Yun Lee São Paulo – SP Menção HonrosaKiyoshi Horie Filho Ourinho – SP Menção HonrosaFábio Eiji Arimura São Paulo – SP Menção HonrosaToni Chenson Wang São Paulo – SP Menção HonrosaGuilherme Tosi Nova Venécia – ES Menção HonrosaYuri Gomes Lima Fortaleza – CE Menção HonrosaThiago Mizuta São Paulo – SP Menção HonrosaLucas Sáber Rocha Macaé – RJ Menção HonrosaDaniel Nascimento Duplat Salvador – BA Menção HonrosaTiago Monteiro Fernandes Rio Claro – SP Menção HonrosaCaio Ribeiro de Souza Rio de Janeiro – RJ Menção HonrosaAdalberto Studart Neto Fortaleza – CE Menção HonrosaRenato Araújo Barbosa Sete Lagoas – MG Menção HonrosaCibele Ferreira de Souza Mineiros – GO Menção HonrosaMarina Lima Medeiros Fortaleza – CE Menção HonrosaCarolina Nunes Nery Belo Horizonte – MG Menção HonrosaGermanna Oliveira Queiroz Fortaleza – CE Menção HonrosaLuciana Akemi Nishimaru São Paulo – SP Menção HonrosaFabiano Siggelkow Linhares São Paulo – SP Menção HonrosaSandraTie Nishibe Minamoto Mogi das Cruzes – SP Menção HonrosaDaniel Haanwinckel Junqueira Salvador – BA Menção HonrosaSolleon Natus Tavares de Menezes Fortaleza – CE Menção HonrosaAnna Laura Sfredo São Paulo – SP Menção HonrosaBruno Gomes Coelho São Paulo – SP Menção HonrosaDaniel Bréscia dos Reis Belo Horizonte – MG Menção HonrosaAndré Bastos Veras Teresina – PI Menção HonrosaLincolnYoshyiti Hamaji São Paulo – SP Menção HonrosaJoão Paulo Aguiar Santos Juiz de Fora – MG Menção HonrosaBruno Bozon Furlan São Paulo – SP Menção HonrosaMárcio Antonio Ferreira Belo Filho Goiânia – GO Menção HonrosaJoão Felipe Almeida Destri Florianópolis – SC Menção HonrosaCaio Bória de Oliveira S. J. dos Campos – SP Menção HonrosaPatrick Gonçalves Jaguaré – ES Menção HonrosaLarissa Goulart Rodrigues Goiânia – GO Menção HonrosaEduardo Horai São Paulo – SP Menção Honrosa

Resultado - Terceiro Nível (Ensino Médio)

NOME CIDADE – ESTADO PRÊMIODaniel Massaki Yamamoto São Paulo – SP OuroDaniel Nobuo Uno São Paulo – SP OuroUlisses Medeiros Albuquerque Fortaleza – CE OuroHumberto Silva Naves Goiânia – GO OuroCarlos Stein Naves de Brito Goiânia – GO PrataLucas Heitzmann Gabrielli São Paulo – SP PrataFabrício Siqueira Benevides Fortaleza – CE PrataGiuliano Boava Criciúma – SC PrataJônathas Diógenes Castello Branco Fortaleza – CE PrataRonaldo Ikaro Farias Araújo Fortaleza – CE PrataCarlos Emanuel Rodrigues Nogueira Fortaleza – CE BronzeDaniel Mourão Martins Fortaleza – CE BronzeGilberto Santos do Nascimento São Paulo – SP BronzeRogério Uhlmann Yamauti São Paulo – SP BronzeFernando Silva Barros C. Lafaiete – MG BronzeLeandro dos Santos de Jesus Rio de Janeiro – RJ BronzeHugo Pinto Iwata S. José do Rio Preto – SP BronzeLeandro de Mattos Ferreira Rio de Janeiro – RJ BronzeBruno Fernandes Cerqueira Leite São Paulo – SP BronzeAdenilson Pereira Bonfim Belém – PA BronzeMônica Mitiko Soares Matsumoto São Paulo – SP BronzeLeonardo da Costa Linhares Rio de Janeiro – RJ BronzeTertuliano Franco Santos Franco Salvador – BA BronzeArthur Duarte Nehmi São Paulo – SP BronzePaulo César de Melo Hanaoka Campo Grande – MS BronzeJoão Alfredo Castellani F. Freire Salvador – BA Menção HonrosaEduardo Famini Silva Salvador – BA Menção Honrosa

EUREKA! N°7, 2000

37

Page 38: CONTEÚDO · Web viewA OBM divulga apenas os nomes e pontuações dos alunos premiados; a OBM nunca divulgou nem divulgará comparações entre colégios. Nosso objetivo é estimular

Sociedade Brasileira de Matemática

Lívia Camargos Rodrigues Oliveira Belo Horizonte – MG Menção HonrosaRoberto Tiburcio Canito Frota Fortaleza – CE Menção HonrosaRui Facundo Vigelis Fortaleza – CE Menção HonrosaCarlos Yuji Hatae São Paulo – SP Menção HonrosaDaniel Pinheiro Sobreira Fortaleza – CE Menção HonrosaPedro Paulo de Simoni Gouvéia Fortaleza – CE Menção HonrosaThiago Barros Rodrigues Costa Fortaleza – CE Menção HonrosaMauricio Masayuki Honda São Paulo – SP Menção HonrosaChristian Lyoiti Watanabe Itaguaí – RJ Menção HonrosaGuilherme Goettems Schneider São Leopoldo – RS Menção HonrosaDanilo Castello Branco A. Bessa São Paulo – SP Menção HonrosaBruno Woltzenlogel Paleo Piracicaba – SP Menção HonrosaCamila Shirota Piracicaba – SP Menção HonrosaMiriam Ou São Paulo – SP Menção HonrosaDiêgo Veloso Uchôa Teresina – PI Menção HonrosaPedro Ferreira Fortaleza – CE Menção HonrosaCelio Hira São Paulo – SP Menção HonrosaGustavo Maltez Lengler Rio de Janeiro – RJ Menção HonrosaFernando Duarte Menezes Fortaleza – CE Menção HonrosaFernando Prado Rocha Goiânia – GO Menção HonrosaPaulo Henrique Jacob Silva São Paulo – SP Menção HonrosaZhang He São Paulo – SP Menção HonrosaRenato Takamatsu São Paulo – SP Menção HonrosaUlisses Duarte Nehmi São Paulo – SP Menção HonrosaEduardo Moraes de Morais São Paulo – SP Menção HonrosaHumberto Vinhais São Paulo – SP Menção HonrosaIlan Felts Almog São Paulo – SP Menção HonrosaPietro Kreitlon Carolino Salvador – BA Menção HonrosaIvo Almino Gondim Fortaleza – CE Menção Honrosa

EQUAÇÕES DIOFANTINASAntonio Caminha Muniz Neto

Nível Intermediário

Denominaremos equação diofantina (em homenagem ao matemático grego Diofanto de Alexandria) uma equação em números inteiros. Nosso objetivo será estudar dois tipos particulares de equações diofantinas, a equação de Pitágoras e a de Pell, e determinar suas soluções. Também estudaremos o método da descida, que nos permitirá mostrar que algumas equações diofantinas não possuem soluções não triviais, num sentido a ser precisado.

Ternos PitagóricosQueremos estudar as soluções (x, y, z) da equação x y z2 2 2 , com x, y, z inteiros não nulos. Após determinar tais soluções, vamos ver como podemos utilizar as informações obtidas para resolver outras equações em números inteiros. O resultado fundamental é o seguinte

Teorema 1: As soluções (x, y, z) da equação x y z2 2 2 , com x, y, z inteiros não nulos, são dadas por: ( , , ) ( , ( ) , ( ) )x y z uvd u v d u v d 2 2 2 2 2

EUREKA! N°7, 2000

38

Page 39: CONTEÚDO · Web viewA OBM divulga apenas os nomes e pontuações dos alunos premiados; a OBM nunca divulgou nem divulgará comparações entre colégios. Nosso objetivo é estimular

Sociedade Brasileira de Matemática

ou onde d, u, v são inteiros não nulos, com u v, mdc(u, v) = 1 e u e v de paridades distintas.

Prova: Sejam x, y, z inteiros positivos quaisquer satisfazendo a equação acima (os demais casos são análogos), e d o mdc de x e y. Então d 2 divide z2 , e daí d divide z. Existem portanto inteiros não nulos a, b, c, com mdc(a, b) = 1, tais que (x, y, z) = (da, db, dc). Ademais, como

x y z a b c2 2 2 2 2 2 ,basta determinarmos as soluções (a, b, c) da equação, sujeitas à condição mdc(a, b) = 1 (que por sua vez implica mdc(a, c) = 1 e mdc(b, c) = 1).Note agora que, dado um inteiro qualquer t, temos que t 2 deixa resto 0 ou 1 na divisão por 4, quando t for respectivamente par ou ímpar. Assim, se fossem a e b ímpares, teríamos a2 e b2 deixando resto 1 na divisão por 4, e daí c a b2 2 2 deixaria resto 2 quando dividido por 4, o que é um absurdo. Como a e b são primos entre si, não podem ser ambos pares. Há então dois casos: a ímpar e b par, a par e b ímpar. Analisemos o primeiro caso (o segundo é análogo).Se a for ímpar e b par, então c também é ímpar. De a b c2 2 2 obtemos b c a c a2 ( )( ) , e não é difícil concluir que mdc(c – a, c + a) = 2. Podemos

então escrever . Note que são primos entre si.

Mas se o produto de dois naturais primos entre si é um quadrado perfeito, então cada um deles deve ser um quadrado perfeito. Existem então inteiros positivos primos entre si u e v, tais que c a v c a u 2 22 2, , e daí ( , , ) ( , , )a b c u v uv u v 2 2 2 22 .Note ainda que, como u v c2 2 é ímpar, u e v devem ter paridades distintas. Por substituição na equação original, concluímos que os ternos acima são realmente soluções da equação, de modo que nada mais há a fazer.

Vemos então que há uma quantidade infinita de ternos (x, y, z) satisfazendo a equação acima. Por exemplo, fazendo d = v = 1 e u = 2n, n inteiro positivo, obtemos o terno ( , , ) ( , , )x y z n n n 4 4 1 4 12 2

Um terno de inteiros positivos (x, y, z) tais que x y z2 2 2 é denominado um terno Pitagórico, em alusão ao matemático grego Pitágoras e seu famoso teorema sobre triângulos retângulos. De fato, um tal terno (x, y, z) determina um triângulo retângulo de catetos x e y e hipotenusa z inteiros.

EUREKA! N°7, 2000

39

Page 40: CONTEÚDO · Web viewA OBM divulga apenas os nomes e pontuações dos alunos premiados; a OBM nunca divulgou nem divulgará comparações entre colégios. Nosso objetivo é estimular

Sociedade Brasileira de Matemática

Vejamos em que a equação acima pode ajudar na solução de outros problemas. Consideremos a tarefa de determinar as soluções inteiras não nulas da equação x y z2 2 22 , com x y. Em uma qualquer dessas soluções, devemos ter x e y com a mesma paridade, pois caso contrário x y2 2 seria um número ímpar. Assim, existem inteiros a e b tais que

x a b y a b ,

Basta tomarmos a x y 12 ( ) e b x y 1

2 ( ) , notando que x + y e x – y são números pares. Substituindo as expressões acima para x e y na equação original, concluímos que

x y z a b z2 2 2 2 2 22 Mas essa última equação é a nossa já conhecida equação de Pitágoras. Então, de acordo com o teorema acima, podemos escrever

ou

onde d, u, v são inteiros não nulos, com u v, mdc(u, v) = 1 e u e v de paridades distintas.Segue daí que as soluções (x, y, z) de nossa equação são de um dos tipos abaixo, onde d, u, v satisfazem as mesmas condições do teorema acima.

ou

Descida de Fermat e Equações sem SoluçõesAs equações analisadas acima são, em um certo sentido, privilegiadas, pois possuem uma infinidade de soluções. Nosso próximo exemplo será o de uma equação que só admite a solução inteira x = y = z = 0. Ela ilustra um método que pode ser estendido a outras equações, a fim de provar que elas não possuem soluções inteiras não nulas.

Exemplo 1: A equação 3 22 2 2x y z não possui soluções inteiras não nulas.

EUREKA! N°7, 2000

40

Page 41: CONTEÚDO · Web viewA OBM divulga apenas os nomes e pontuações dos alunos premiados; a OBM nunca divulgou nem divulgará comparações entre colégios. Nosso objetivo é estimular

Sociedade Brasileira de Matemática

Prova: Suponha o contrário. Então a equação possui uma solução (x, y, z) em inteiros positivos. Então, dentre todas as soluções (x, y, z), com x, y e z inteiros positivos, existe uma (x, y, z) = (a, b, c) para a qual z = c é o menor possível. Trabalhemos tal solução.

Vamos usar o seguinte fato, que você pode provar facilmente: se um inteiro u não for múltiplo de 3, então u2 deixa resto 1, quando dividido por 3. Então, se b não for múltiplo de 3, teremos de 3 22 2 2a b c que c também não será múltiplo de 3. Olhando os restos de cada termo da equação por 3, teremos que 3 2 2a b deixa resto 1 e 2 2c deixa resto 2.1 = 2. Logo, não poderia ser 3 22 2 2a b c . Assim, b deve ser múltiplo de 3, digamos b b3 1 . Daí vem que 3 9 22

12 2a b c ,

e c também é múltiplo de 3, digamos c c3 1 . Substituindo na equação, chegamos a 3 61

2 212b a c .

Então, a também é múltiplo de 3. Sendo a a3 1 , a equação acima nos dá b a c1

212

123 2 , e ( , , )b a c1 1 1 é uma outra solução de nossa equação original,

com c cc1 3 . Mas isso é uma contradição, pois partimos de uma solução na

qual o valor de z era c, mínimo possível. Logo, nossa equação não possui soluções não nulas.

Esquematicamente, o método da descida (devido ao matemático francês Pierre Simon de Fermat) consiste então no seguinte:

i. Supor que uma dada equação possui uma solução em inteiros não nulos.ii. Concluir daí que ela possui uma solução em inteiros positivos que seja, em

algum sentido, mínima.iii. Deduzir a existência de uma solução positiva menor que a mínima,

chegando a uma contradição.

Já que determinamos acima as soluções da equação de Pitágoras, nada mais natural que tentar estudar a equação mais geral abaixo, denominada equação de Fermat. Aqui, n > 2 é um inteiro fixado.

x y zn n n ,Por cerca de três séculos os matemáticos defrontaram-se com o problema de decidir sobre a existência de soluções não nulas (x, y, z) dessa equação, problema que somente foi resolvido na década de noventa, utilizando métodos muitíssimo complexos.

EUREKA! N°7, 2000

41

Page 42: CONTEÚDO · Web viewA OBM divulga apenas os nomes e pontuações dos alunos premiados; a OBM nunca divulgou nem divulgará comparações entre colégios. Nosso objetivo é estimular

Sociedade Brasileira de Matemática

Vamos aproveitar o método da descida para analisar um caso simples dessa equação, aquele em que n é um múltiplo de 4. O leitor interessado em saber mais sobre essa equação pode consultar uma das referências [2] ou [3] da bibliografia, onde o caso n = 3 é discutido.

Teorema 2: Se n for múltiplo de 4 então não existem inteiros não nulos x, y, z tais que x y zn n n .

Prova: Seja n = 4k, k natural. Se x y zn n n , então teremos ( ) ( ) ( )x y zk k k4 4 2 2 , ou seja, ( , , )x y zk k k2 será uma solução da equação a b c4 4 2 . Assim, basta mostrarmos que essa última equação não admite soluções não nulas. Por absurdo, suponhamos que existam inteiros positivos a, b, c tais que a b c4 4 2 . Podemos também supor que a, b e c foram escolhidos de tal modo que não há outra solução positiva a b c' , ' , ' com (aqui vamos usar o método da descida). Então a e b são primos entre si, e o teorema 1 garante a existência de inteiros positivos primos entre si u e v tais que a u v b uv c u v2 2 2 2 2 22 , , . Como a v u2 2 2 , segue novamente do teorema 1 a existência de inteiros positivos primos entre si p e q tais que a p q v pq u p q 2 2 2 22, , . Mas aí

Como p e q são primos entre si, temos que ambos são também primos com p q2 2 . Portanto, sendo 4 2 2pq p q( ) um quadrado devemos ter p, q e p q2 2 quadrados, digamos p q p q 2 2 2 2 2, , , com , , positivos. Por fim, segue que 4 4 2 , com contrariando a minimalidade de c. Logo, não há soluções não nulas de x y zn n n quando n for múltiplo de 4.

A Equação de PellNem sempre é fácil, ou mesmo possível, determinar todas as soluções em inteiros de uma dada equação. Por exemplo, para a equação x y2 22 1 , é bem mais fácil mostrar que ela possui uma infinidade de soluções do que determinar todas elas. Podemos gerar infinitas soluções dessa equação a partir de uma só solução não nula.Uma vez que a b2 22 1 , teremos ( )( )a b a b 2 2 1 , e daí

( ) ( )a b a b 2 2 12 2

Desenvolvendo os binômios, chegamos a, e daí a

EUREKA! N°7, 2000

42

Page 43: CONTEÚDO · Web viewA OBM divulga apenas os nomes e pontuações dos alunos premiados; a OBM nunca divulgou nem divulgará comparações entre colégios. Nosso objetivo é estimular

Sociedade Brasileira de Matemática

Portanto, se (a, b) for uma solução, será outra solução. Sendo a e b positivos, temos , e desse modo determinamos uma infinidade de soluções da equação (contanto que tenhamos uma solução não nula). Veja que (3, 2) é uma solução não nula de nossa equação.É fácil ver que o método acima utilizado também garante que, quando d for um inteiro tal que d é irracional, a equação x dy2 2 1 admite infinitas soluções não nulas, desde que admita uma solução não nula. Também, com poucas modificações podemos tratar a equação x dy2 2 1 (veja o exercício 6). Observe que, apesar de determinarmos facilmente infinitas soluções da equação acima, não sabemos se há outras. Vamos agora começar a responder essa pergunta, para uma classe mais ampla de equações.

Definição 1 (Equação de Pell): Seja d um inteiro positivo que não seja um quadrado. Nesse caso, sabemos que d é irracional. Chamamos equação de Pell à equação x dy m2 2 , onde m é um inteiro qualquer.É claro que no caso m = 0 a equação não admite soluções além da trivial x = y = 0, pois se esse fosse o caso teríamos x e y não nulos, e daí d x

y , um racional.Lema 1: Seja um irracional qualquer. Existem infinitos racionais x

y , com x e y

inteiros não nulos primos entre si, tais que xy y 1

2 .Prova: Seja n > 1 um natural qualquer, e considere os números j , com j = 0, 1, ..., n. Seja Como

0 1 0 11 1 2 1, , , ... , n n n

nn ,

segue do princípio de Dirichlet que existem 0 k j n tais que { }j e { }k pertencem a um mesmo intervalo dos que aparecem no lado direito da igualdade acima. Então |{ }j - { }k | < 1

n Daí, j k j k n 1 , e segue que

Existe então um par (x, y) de inteiros, x j k y j k n , , tais que xy y 1

2 . Se x dx y dy 1 1, , com d > 1, então xy y y11

12

1

12 , de modo

que podemos supor que x e y são primos entre si.Para garantirmos a existência de infinitos tais pares, suponha que achamos x e y primos entre si e tais que x

y y 1

2 . Escolha agora um natural n tal que

EUREKA! N°7, 2000

43

Page 44: CONTEÚDO · Web viewA OBM divulga apenas os nomes e pontuações dos alunos premiados; a OBM nunca divulgou nem divulgará comparações entre colégios. Nosso objetivo é estimular

Sociedade Brasileira de Matemática

1n

xy . Repetindo o argumento acima, chegamos a um par de inteiros primos

entre si x y1 1, , com e . Portanto,

e , donde

( , ) ( , )x y x y1 1 satisfaz o lema.

Lema 2: Seja d um inteiro positivo que não seja um quadrado. Existe um inteiro m para o qual a equação x dy m2 2 admite infinitas soluções inteiras.

Prova: Sabemos que d é irracional. Assim, o conjunto S dos pares (x, y) de

inteiros primos entre si tais que xy y

d 12 é infinito. Mas se x e y forem

inteiros satisfazendo essa desigualdade, então

Segue que algum inteiro não nulo m entre ( )2 1d e 2 1d se repete um número infinito de vezes entre os valores de x dy2 2 , com (x, y) em S. Mas isto é o mesmo que dizer que a equação x dy m2 2 admite infinitas soluções.

Teorema 3 (Soluções da Equação de Pell): Seja d um inteiro positivo que não seja um quadrado. A equação x dy2 2 1 admite infinitas soluções em inteiros positivos x, y. Ademais, existe uma solução em inteiros positivos x y1 1, tal que todas as demais soluções dessa equação são da forma x y d x y dn n

n ( )1 1 , onde n é um número natural.

Prova: Admitamos por enquanto que nossa equação tenha uma solução em inteiros positivos x, y. Dentre todas essas soluções, escolha aquela x y1 1, tal que x y d1 1 seja o menor possível. Dado um natural qualquer n, sabemos que existem inteiros positivos x yn n, tais que ( )x y d x y dn

n n1 1 . Daí, sabemos que

( )x y d x y dnn n1 1 , e assim

1 12

12

1 1 1 12 2

( ) ( ) ( )

( )( )

x dy x y d x y d

x y d x y d x dy

n n n

n n n n n n

EUREKA! N°7, 2000

44

Page 45: CONTEÚDO · Web viewA OBM divulga apenas os nomes e pontuações dos alunos premiados; a OBM nunca divulgou nem divulgará comparações entre colégios. Nosso objetivo é estimular

Sociedade Brasileira de Matemática

Então todos os pares ( , )x yn n são soluções da equação. Seja agora (x, y) uma solução qualquer em inteiros positivos. Para terminar, basta mostrarmos que existe um natural n tal que x y d n . Suponha o contrário. Então existe um natural n tal que n nx y d 1 . Daí, vem que 1 n x y d( ) . Mas

n nn n

n n n n n n

x y d x y d x y d x y d x y d

x y d x y d xx dyy x y y x d

( ) ( ) ( ) ( ) ( )

( )( ) ( ) ( )1 1

1

e ocorre que ,

de modo que nn n n nx y d xx dyy x y y x( ) ( , ) também é solução.

Como , basta mostrarmos que para chegarmos numa contradição. Sejam a xx dyy b x y y xn n n n , . Temos a b d 0 e a db2 2 1 , donde a b d a b d ( ) 1 0 .Então, 2 0a a b d a b d ( ) ( ) . Por outro lado, a b d 1 implica a b d a b d ( ) 1 1 , e daí b d a 1 0 . Logo, b > 0.Para terminar, basta mostrarmos que a equação x dy2 2 1 admite uma solução. Tome, de acordo com o lema 2, um inteiro (não nulo) m tal que x dy m2 2 admita uma infinidade de soluções. Podemos escolher duas dessas soluções, ( , ), ( , )x y x y1 1 2 2 digamos, tais que | | | |x x1 2 mas x x1 2 e y y1 2 , módulo m. Então

( )( ) ( ) ( )x y d x y d x x dy y x y x y d1 1 2 2 1 2 1 2 2 1 1 2 (*)Mas x x dy y x dy m1 2 1 2 1

212 0 (mod ) e x y x y m2 1 1 2 (mod ) , donde existem

inteiros u e v tais que x x dy y mu x y x y mv1 2 1 2 2 1 1 2 ,Segue de (*) que ( )( ) ( )x y d x y d m u v d1 1 2 2 , e daí

( )( ) ( )x y d x y d m u v d1 1 2 2 .Multiplicando ordenadamente essas duas igualdades, chegamos a

m x dy x dy m u dv212

12

22

22 2 2 2 ( )( ) ( ) ,

ou seja, u dv2 2 1 . Resta mostrarmos que u e v são não nulos. Se u = 0 teríamos dv2 1 , um absurdo. Se v = 0, viria u = 1 ou –1. De (*) seguiria que ( )( )x y d x y d m1 1 2 2 , e assim ( ) ( )x y d x y d1 1 2 2 , donde por fim | | | |x x1 2 , o que é um absurdo.

EUREKA! N°7, 2000

45

Page 46: CONTEÚDO · Web viewA OBM divulga apenas os nomes e pontuações dos alunos premiados; a OBM nunca divulgou nem divulgará comparações entre colégios. Nosso objetivo é estimular

Sociedade Brasileira de Matemática

Exemplo 2: Agora podemos determinar todas as soluções inteiras não nulas da equação x y2 22 1 . O teorema 3 ensina que as soluções positivas dessa equação são da forma ( , )x yn n , onde xn e yn são os únicos inteiros para os quais x y x yn n

n 2 21 1( ) , sendo ( , )x y1 1 a solução positiva para a qual x y1 1 2 é o menor possível. Como os pares (x, y) = (1, 1), (1, 2), (2, 1), (2, 2), (2, 3) não são soluções da equação e (3, 2) é, é fácil nos convencermos de que ( , )x y1 1 = (3, 2). Desse modo, temos os pares ( , )x yn n dados pela igualdadex yn n

n 2 3 2 2( )Determine agora as demais soluções não nulas da equação acima. O exercício 7 discute mais alguns aspectos dessa equação.

Exercícios:1. Seguindo os passos da prova do teorema 1, mostre que as soluções eminteiros não nulos da equação x y z2 2 22 são da forma x u v d y uvd z u v d ( ) , , ( )2 2 2 22 2 2 , onde d, u, v são inteirosnão nulos, com u e 2v primos entre si.2. Mostre que as equações a seguir não possuem soluções inteiras não nulas:i. x y z4 4 24

ii. x y z4 4 22

iii. x y z2 2 23 O item i do exercício a seguir tem a ver com o exemplo 1 do texto.3. i. Mostre que não existem racionais x e y tais que x xy y2 2 2 .ii. Determine todas as soluções racionais da equação x xy y2 2 1 .Para resolver os próximos dois exercícios utilizamos o teorema 1. Eles são mais difíceis que os anteriores, e no primeiro deles você pode achar útil o seguinte resultado, conhecido como Teorema de Ptolomeu: dado um quadrilátero convexo inscritível ABCD, tem-se

AB CD AD BC AC BD. . .

EUREKA! N°7, 2000

46

Page 47: CONTEÚDO · Web viewA OBM divulga apenas os nomes e pontuações dos alunos premiados; a OBM nunca divulgou nem divulgará comparações entre colégios. Nosso objetivo é estimular

Sociedade Brasileira de Matemática

Para uma prova do Teorema de Ptolomeu, você pode consultar a referência [4].4. Temos no plano uma circunferência de raio 1. Mostre que podemosescolher em tal circunferência 2000 pontos A A A1 2 2000, , ..., tais que A Ai j é racional, quaisquer que sejam 1 2000 i j .5. Seja r um inteiro positivo dado. Queremos determinar o número detriângulos ABC, dois a dois não congruentes, satisfazendo as seguintes condições:i. O raio da circunferência inscrita em ABC mede r.ii. Os comprimentos dos lados de ABC são números inteiros, primos entre si.Mostre que o número de tais triângulos é 2k , onde k é o número de fatores primos distintos de r.6. Prove, sem apelar para o teorema 2, que a equação x y2 22 1 admite uma infinidade de soluções inteiras.7. Prove que as soluções positivas ( , )x yn n da equação do exemplo 2 sãodadas pelas seqüências ( , ) ( , )x y1 1 3 2 e , 8. Prove que há infinitos inteiros n tais que seja quadrado.

Bibliografia

[1] Introdução à Teoria dos Números. Plínio O. dos Santos. Coleção Matemática Universitária.IMPA. 1999. [2] An Introduction to the Theory of Numbers. I. Niven, H. Zuckermann. John Wiley & Sons. New York. 1980.[3] A Classical Introduction to Modern Number Theory. K. Ireland & M. Rosen. Springer-Verlag.New York. 1990. [4] Quadriláteros e Triângulos. M. Mendes. Eureka! No5. OBM 1999

SOLUÇÕES DE PROBLEMAS PROPOSTOS

EUREKA! N°7, 2000

47

Page 48: CONTEÚDO · Web viewA OBM divulga apenas os nomes e pontuações dos alunos premiados; a OBM nunca divulgou nem divulgará comparações entre colégios. Nosso objetivo é estimular

Sociedade Brasileira de Matemática

Publicamos aqui algumas das respostas enviadas por nossos leitores.

29) Seja n > 1 um número inteiro. Existem n lâmpadas L0, L1, ... , Ln–1

colocadas em um círculo. Cada lâmpada está ACESA ou APAGADA. Uma seqüência de passos S0, S1, ... , Si, ... é executada. O passo Sj afeta apenas o estado da lâmpada Lj (deixando o estado de todas as outras inalterado) da seguinte forma:

Se Lj–1 está ACESA, Sj muda o estado de Lj de ACESA para APAGADA, ou de APAGADA para ACESA;

Se Lj–1 está APAGADA, Sj deixa o estado de Lj inalterado.

As lâmpadas são rotuladas mod n, ou seja, L – 1 = Ln –1, L0 = Ln, L1 = Ln + 1, etc.

Inicialmente todas as lâmpadas estão ACESAS. Mostre que:

a) Existe um inteiro positivo M(n) tal que depois de M(n) passos todas as lâmpadas estão ACESAS de novo;

b) Se n é da forma 2k então todas as lâmpadas estão ACESAS depois de n2 –1 passos;

c) Se n tem a forma 2k + 1 então todas as lâmpadas estão ACESAS depois de n2 – n + 1 passos.

Solução de Frank Castro (São Paulo - SP):

a) Vamos inicialmente representar o estado das lâmpadas L0, L1, L2, ..., Ln–1 por uma n-upla u = (u0, u1, u2, ..., un–1), onde ui = 0 se Li está apagada e ui = 1 se Li

está acesa.Evidentemente o estado inicial das lâmpadas é dado pela n-upla e = (1, 1, 1, ...,1). Nessas condições a operação Sj tranforma a n-upla (u0, u1, u2, ..., un–1) na nova n-upla (u0, u1,..., uj–1, uj–1 + uj,..., uj+1,..., un–1), onde a soma uj–1 + uj é tomada módulo 2 ( e j é tomada módulo n).Assim sendo, nosso problema consiste em determinar um valor natural r, tal que:

Para tanto, denotemos por Rj a operação que transforma a n-upla (u0, u1,..., uj–1, uj, uj+1,..., un–1) na n-upla Observe que R–j é a operação inversa de Rj e R0 deixa a n-upla inalterada.

EUREKA! N°7, 2000

48

Page 49: CONTEÚDO · Web viewA OBM divulga apenas os nomes e pontuações dos alunos premiados; a OBM nunca divulgou nem divulgará comparações entre colégios. Nosso objetivo é estimular

Sociedade Brasileira de Matemática

Nesses termos temos para uma n-upla qualquer .

Agora para uma n-upla qualquer podemos escrever:

Consequentemente, como existe apenas um número finito de estados das lâmpadas (2n precisamente) que equivale ao número total de n-uplas, em algum estágio a seqüência

deve repetir algum de seus elementos. Nessas condições para algum m e n ( m < n ) teremos : Sendo

uma bijeção (verifique!) temos o que conclui o ítem a.

b) Primeiramente associaremos à n-upla u = (u0, u1, u2, ..., un–1) o polinômio P(x) da forma : onde os coeficientes serão olhados módulo 2 (isto é, P(x) Chamemos tal polinômio de polinômio de posição. Observe agora que para a n-upla temos:

e seu polinômio de posição é dado por Não se esqueça

que a adição é tomada módulo 2. Assim, Assim sendo, queremos encontrar r tal que

Note que quando o polinômio de posição associado a será congruente a P(x) que representará a n-upla e = (1, 1, 1,...1) no estado inicial das lâmpadas. Suponha agora n = 2k. Então pois se n é uma potência de 2, todos os coeficientes, exceto o primeiro e o último da expansão binomial são pares, logo congruentes a zero módulo 2.Finalmente temos:

EUREKA! N°7, 2000

49

Page 50: CONTEÚDO · Web viewA OBM divulga apenas os nomes e pontuações dos alunos premiados; a OBM nunca divulgou nem divulgará comparações entre colégios. Nosso objetivo é estimular

Sociedade Brasileira de Matemática

Assim, após etapas todas as lâmpadas estarão acesas novamente.c) Suponha Assim onde todas as congruências foram tomadas Como no ítem anterior n – 1 é potência de 2, logo todos os coeficientes, exceto o primeiro e o último da expansão binômial

são pares, consequentemente congruentes a zero módulo 2.Finalmente temos:

Observe que, como estamos trabalhando módulo 2, e isso justifica a congruência (*). Assim sendo, após etapas, todas as lâmpadas estarão acesas novamente.

30) Determine todas as funções f : R R que satisfazem as condições:

(i) f(– x) = – f(x), (ii) f(x + 1) = f(x) + 1, (iii) para x 0.

Solução de Carlos Alberto da Silva Victor (Rio de Janeiro - RJ):

Do ítem (ii) : f(x + 1) = f(x) temos que f(x) = 1 + f( x – 1) e usando (i)

f(x) + f (1 – x) = 1. Sejam x 0 e x 1, logo: e usando (iii)

e usando novamente (iii)

EUREKA! N°7, 2000

50

Page 51: CONTEÚDO · Web viewA OBM divulga apenas os nomes e pontuações dos alunos premiados; a OBM nunca divulgou nem divulgará comparações entre colégios. Nosso objetivo é estimular

Sociedade Brasileira de Matemática

donde: para

Conclusão: x R, teremos f(x) = x como sendo a única solução.

31) Seja x1, x2, x3, … uma seqüência de números reais não negativos satisfazendo

para n = 3, 4, 5, … Estabeleça condições necessárias e

suficientes em x1 e x2 para xn ser inteiro para infinitos valores de n.

Solução de Davi Máximo Alexandrino Nogueira (Fortaleza - CE):

Afirmação: inteiros.

Prova: Se, teremos,

tome logo, a seqüência y1, y2, …, é uma

P.A., de razão desse modo,

Suponha r 0.

Como , temos fazendo x1 – x2 = a, teremos

EUREKA! N°7, 2000

51

Page 52: CONTEÚDO · Web viewA OBM divulga apenas os nomes e pontuações dos alunos premiados; a OBM nunca divulgou nem divulgará comparações entre colégios. Nosso objetivo é estimular

Sociedade Brasileira de Matemática

. Porém, para algum k, tal que teremos

, para todo n k. Logo, devemos ter r = 0, o que conclui a demonstração.

32) a) Prove que todo número inteiro não nulo m admite uma única representação

da forma onde n é um inteiro positivo e para todo

k, com

Dado um conjunto de pontos , escrevemos em cada

aresta que une dois desses pontos Pi e Pj (i j) um número pertencente a

{0, 1, …, n – 1} da seguinte forma: escreveremos com

e associamos à aresta Pi Pj o número

Prove que não existe nenhum triângulo cujos vértices pertençam a V com o mesmo número escrito em seus três lados.

Solução de Carlos Alberto da Silva Victor (Rio de Janeiro - RJ):

a) Sabendo que m 0 (mod 3) ; m 1 (mod 3) ou m – 1 (mod 3), teremos:

m = 3k0 + r0

k0 = 3k1 +r1

k1 = 3k2 + r2kn–2 = 3kn–1 + rn–1

com kn–1 = 0 e rn–1 0, onde ri {–1, 0, 1} (note que os ri estão unicamente determinados, e existe n com kn–1 = 0 pois enquanto tivermos ) Substituindo na primeira igualdade k0 pela sua igualdade, obteremos m como função de k1, r0. Tomando novamente k1 = 3 k2 + r2 e fazendo as substituições sucesivas de k2, k3, ..., kn-2 ; obtemos:

EUREKA! N°7, 2000

52

Page 53: CONTEÚDO · Web viewA OBM divulga apenas os nomes e pontuações dos alunos premiados; a OBM nunca divulgou nem divulgará comparações entre colégios. Nosso objetivo é estimular

Sociedade Brasileira de Matemática

e já que ,

teremos com

b) Sejam três pontos quaisquer de V e por hipótese i < j < s;

portanto podemos escrever: e , com

Vamos também fazer a hipótese de que para as arestas tenhamos

o número onde

Podemos então escrever:

De (2) – (1) obtemos

Para pertencem a {–1, 0}, donde e

os somatórios com são múltiplos de , e portanto, ao

escrever , com não aparece o termo , o que

garante que o número associado à aresta Pj Ps, que é em não será igual a .

Conclusão: não existe triângulo cujos vértices pertençam a V com o mesmo número escrito em seus três lados.33) Na parede interna de um vaso cilíndrico de cristal existe uma gota de melnum ponto B situado a três centímetros do seu bordo superior. Na parede externa, num ponto A diametralmente oposto ao da gota, está uma formiga. Sabendo que a altura do vaso é de 20cm e o seu diâmetro é 10cm. Indicar o caminho mais curto para que a formiga atinja a gota de mel.

Solução de Daniel Pessôa Martins Cunha (Fortaleza - CE):

EUREKA! N°7, 2000

53

Page 54: CONTEÚDO · Web viewA OBM divulga apenas os nomes e pontuações dos alunos premiados; a OBM nunca divulgou nem divulgará comparações entre colégios. Nosso objetivo é estimular

Sociedade Brasileira de Matemática

Cobrindo o vaso com papel por dentro e por fora, e marcando nele a localização da formiga, da gota de mel e da borda, poderemos ver que ao desamassar o papel ficarão as seguintes impressões, com as seguintes medidas:

Como a menor distância entre 2 pontos é a medida do segmento que os une, o menor caminho é o segmento de medida : (Teorema de Pitágoras).Ao colocar o papel de volta ao vaso veremos o menor caminho a ser percorrido pela formiga. Que é subir em diagonal até o ponto médio do arco , determinado pelo diâmetro na borda. Depois descer em diagonal até a gota de mel.

34) ABC é um triângulo, tal que AB = c, AC = b e BC = a.Por um ponto interior P deste triângulo, são traçadas paralelas aos seus lados.Sabe-se que as intersecções, da paralela ao lado de medida a, com os lados deste triângulo, determinam um segmento de medida a'.Analogamente, as paralelas aos lados de medidas b e c, determinam com os lados do triângulo, segmentos de medidas b' e c' respectivamente.

Nestas condições demonstre que

EUREKA! N°7, 2000

54

Page 55: CONTEÚDO · Web viewA OBM divulga apenas os nomes e pontuações dos alunos premiados; a OBM nunca divulgou nem divulgará comparações entre colégios. Nosso objetivo é estimular

Sociedade Brasileira de Matemática

Solução de Francisco Antonio Martins de Paiva (Fortaleza - CE):

De acordo com o problema temos:

Como as retas traçadas são paralelas aos lados então os quadriláteros PFCE, PIBD, PHAG são paralelogramos, e com isso concluímos que seus lados opostos são congruentes. Daí temos:BC = a, AC = b, AB = c, IF = x + y = a' , GD = t + w = c', HE = s + z = b'Os triângulos ABC e AIF são semelhantes pois , de onde temos:

e

Os triângulos ACB e GCD são semelhantes pois , de donde temos:

e

Os triângulos ACB e HEB são semelhantes pois , de onde temos:

EUREKA! N°7, 2000

55

Page 56: CONTEÚDO · Web viewA OBM divulga apenas os nomes e pontuações dos alunos premiados; a OBM nunca divulgou nem divulgará comparações entre colégios. Nosso objetivo é estimular

Sociedade Brasileira de Matemática

e

Daí temos que:

35) Sabendo que num triângulo ABC a altura relativa ao vértice A mede 12cm. e aaltura relativa ao vértice B mede 20cm, determine todos os valores possíveis para a altura relativa ao vértice C.

Solução de Frank Castro (São Paulo - SP):

Temos: ha = 12, hb = 20.Sendo a, b e c os lados do triângulo e S sua área, valem as seguintes relações:

(I)

e Substituindo as três últimas igualdades em (I)

Vem que:

assim hc = 30. Agora, sabemos que: a + b > c

EUREKA! N°7, 2000

56

Page 57: CONTEÚDO · Web viewA OBM divulga apenas os nomes e pontuações dos alunos premiados; a OBM nunca divulgou nem divulgará comparações entre colégios. Nosso objetivo é estimular

Sociedade Brasileira de Matemática

e . Substituindo temos:

Resposta: 7,5cm < hc < 30 cm.

Agradecemos também o envio das soluções a: Ricardo Klein Hoffmann (Porto Alegre - RS), Geraldo Perlino Júnior (São Paulo - SP), José Heleno Faro (Cachoeiro de Itapemirim - ES).

Você sabia…Que há novos records de primos grandes descobertos em 2000?

Maior par de primos gêmeos conhecido: 2409110779845260000±1.Esses primos têm 18075 dígitos, e foram descobertos por Wassing, Járai e Indlekofer.

Maior primo de Fermat generalizado conhecido: 16717632768 + 1, que tem 171153 dígitos e foi descoberto por Yves Gallot (este é o oitavo maior primo conhecido atualmente, e maior primo conhecido que não é de Mersenne).

PROBLEMAS PROPOSTOS

Convidamos o leitor a enviar soluções dos problemas propostos e sugestões de novos problemas para os próximos números.

36) Na figura abaixo o triângulo DEF tem área de medida S. Sabendo-se que o triângulo DEF está inscrito num triângulo arbitrário ABC, mostre que as medidas Si ( i = 1, 2, 3) das áreas dos outros triângulos formados satisfazem a

desigualdade e que a igualdade ocorre se e só se os pontos

DEF são os pontos médios dos lados do triângulo, ABC.

EUREKA! N°7, 2000

57

Page 58: CONTEÚDO · Web viewA OBM divulga apenas os nomes e pontuações dos alunos premiados; a OBM nunca divulgou nem divulgará comparações entre colégios. Nosso objetivo é estimular

Sociedade Brasileira de Matemática

37) Cinco quadrados são dispostos conforme ilustra o diagrama abaixo. Mostre que a medida da área do quadrado S é igual a medida da área do triângulo T.

38) Os lados e diagonais de um polígono regular de n lados são coloridos em k cores tais que:

i) para cada cor a e dois vértices A e B do polígono, o segmento AB é colorido de a ou existe um vértice C tal que AC e BC são coloridos de a.

ii) os lados de qualquer triângulo com vértices entre os vértices do polígono são coloridos usando no máximo 2 cores. Prove que k 2.

39) Sejam x, y e z os ângulos de um triângulo de lados opostos a, b e c respectivamente. Prove que,

40) a) Calcular a soma dos divisores positivos de um número natural em termos de sua fatoração prima.b) Dizemos que n 1 é abundante se a soma de seus divisores é maior que 2n. Prove que se n é abundante então kn é abundante para todo inteiro k 1.

EUREKA! N°7, 2000

58

Page 59: CONTEÚDO · Web viewA OBM divulga apenas os nomes e pontuações dos alunos premiados; a OBM nunca divulgou nem divulgará comparações entre colégios. Nosso objetivo é estimular

Sociedade Brasileira de Matemática

c) Prove que existe n0 N tal que todo inteiro n n0 pode ser escrito como soma de dois números abundantes.

Problemas 36 e 37 propostos por Carlos Alexandre Gomes da Silva (Natal - RN), problema 38 proposto na Olimpíada Búlgara - 1998, problema 39 proposto por Aldo Trajano Louredo, problema 40 proposto por Gleydson Chaves Ricarte (Fortaleza - CE) e Zoroastro Azambuja Neto (Rio de Janeiro - RJ).

Errata:

Eureka!No. 6, pág 40: O enunciado do problema 4 deve dizer:Problema 4: Mostre que há infinitos naturais n tais que divide n!, onde n! = n (n–1)..21 (por exemplo, 4! = 4 3 2 1 = 24).

Eureka! No. 6, pág 27: o segundo parágrafo está truncado. A versão correta é:Determinar exatamente os valores de números de Ramsey clássicos R(a, b) é, em geral, um problema computacionalmente muito difícil. Os únicos valores de R(a, b) com 3 a b que são conhecidos são: R(3, 3) = 6, R(3, 4) = 9, R(3, 5) = 14, R (3, 6) = 18, R(3, 7) = 23, R(3, 8) = 28, R (3, 9) = 36, R(4, 4) = 18, e R(4, 5) = 25. O único número Ramsey com mais de duas cores cujo valor é conhecido é R(3, 3, 3) = 17.

AGENDA OLÍMPICA

XI OLIMPÍADA DO CONE SUL14 a 19 de abril de 2000Montevideu – Uruguai

VI OLIMPÍADA DE MAIO13 de maio de 2000

EUREKA! N°7, 2000

59

Page 60: CONTEÚDO · Web viewA OBM divulga apenas os nomes e pontuações dos alunos premiados; a OBM nunca divulgou nem divulgará comparações entre colégios. Nosso objetivo é estimular

Sociedade Brasileira de Matemática

XXII OLIMPÍADA BRASILEIRA DE MATEMÁTICAPrimeira Fase – Sábado, 10 de junho

Segunda Fase – Sábado, 02 de setembroTerceira Fase – Sábado, 21 de outubro (níveis 1,2 e 3)

Domingo, 22 de outubro (nível 3 - segundo dia).

XLI OLIMPÍADA INTERNACIONAL DE MATEMÁTICA13 a 25 de julho

Taejon, Coreia do Sul.

XV OLIMPÍADA IBEROAMERICANA DE MATEMÁTICA16 a 24 de setembro de 1998

Caracas, Venezuela

III OLIMPÍADA IBEROAMERICANA DE MATEMÁTICA UNIVERSITÁRIAoutubro de 2000

COORDENADORES REGIONAIS

Amarisio da Silva Araújo (UFV) Viçosa - MG Alberto Hassen Raad (UFJF) Juiz de Fora - MGAngela Camargo (Centro de Educ.de Adultos - CEA) Blumenau - SCBenedito T. Vasconcelos Freire (UFRN) Natal - RNClaudio Arconcher (Col. Leonardo da Vinci) Jundiaí - SPClaus Haetinger (UNIVATES) Lajeado - RSCrescêncio das Neves (UFAM) Manaus-AMÉlio Mega (Col. ETAPA) São Paulo - SPEnzo Marcom Takara (Col. Singular) Santo André - SPFlávia Jerônimo Barbosa (UFPB Campus I) João Pessoa - PBFlorêncio F. Guimarães Filho (UFES) Vitória - ESFrancisco Dutenhefner (UFMG) Belo Horizonte - MGGisele de A. Prateado Gusmão (UFGO) Goiânia - GOIvanilde H. Fernandes Saad (U. Católica Dom Bosco) Campo Grande - MS

EUREKA! N°7, 2000

60

Page 61: CONTEÚDO · Web viewA OBM divulga apenas os nomes e pontuações dos alunos premiados; a OBM nunca divulgou nem divulgará comparações entre colégios. Nosso objetivo é estimular

Sociedade Brasileira de Matemática

João Benício de Melo Neto (UFPI) Teresina - PIJoão F. Melo Libonati (Grupo Educ. IDEAL) Belém - PAJorge Ferreira (UEM) Maringá - PRJosé Carlos Pinto Leivas (UFRG) Rio Grande - RSJosé Cloves Saraiva (UFMA) São Luis - MAJosé Gaspar Ruas Filho (ICMC-USP) São Carlos - SPJosé Luis Rosas Pinho (UFSC) Florianópolis - SCJosé Paulo Carneiro (Univ. Santa Úrsula) Rio de Janeiro - RJJosé Vieira Alves (UFPB) Campina Grande - PBLeonardo Matteo D'orio (Sistema Titular de Ensino)Belém - PALicio Hernandes Bezerra (UFSC) Florianópolis - SCLuzinalva M. de Amorim (UFBA) Salvador - BAMarcondes Cavalcante França (UF Ceará) Fortaleza - CEPablo Rodrigo Ganassim (L. Albert Einstein) Piracicaba - SPPaulo H. Cruz Neiva de L. Jr. (Esc. Tec.Everardo Passos) SJ dos Campos - SPReinaldo Gen Ichiro Arakaki (INPE) SJ dos Campos - SPRicardo Amorim (Centro Educ. Logos) Nova Iguaçu - RJRoberto Vizeu Barros (Colégio ACAE) Volta Redonda - RJSergio Claudio Ramos (IM-UFRGS) Porto Alegre - RSSeme Gebara Neto (UFMG) Belo Horizonte - MGSilvio de Barros Melo (UFPE) Recife - PETadeu Ferreira Gomes (U. do Estado da Bahia) Juazeiro - BATomás Menéndez Rodrigues (U. Federal de Rondonia) Porto Velho - ROValdenberg Araújo da Silva (U. Federal de Sergipe) São Cristovão - SEWagner Pereira Lopes (Esc. Tec. Fed. de Goiás) Jataí - GOWaldemar M. Canalli (P.M. S. João de Meriti) S. João de Meriti - RJ

EUREKA! N°7, 2000

61